Jump to content

Wikipedia:Reference desk/Science: Difference between revisions

From Wikipedia, the free encyclopedia
Content deleted Content added
Line 449: Line 449:


So I have a function called any_polynomial which will take an argument of such an array and return the anonymous function, which can be assigned a function handle. This is for an assignment, so I only really want a hint or two. The assignment tells us to use polyval, which is quite useless for this in my opinion, as then the anonymous function will be something like "@(x) polyval(input_array,x)". Thoughts? --[[User:M1ss1ontomars2k4|M1ss1ontomars2k4]] ([[User talk:M1ss1ontomars2k4|talk]]) 03:56, 29 September 2008 (UTC)
So I have a function called any_polynomial which will take an argument of such an array and return the anonymous function, which can be assigned a function handle. This is for an assignment, so I only really want a hint or two. The assignment tells us to use polyval, which is quite useless for this in my opinion, as then the anonymous function will be something like "@(x) polyval(input_array,x)". Thoughts? --[[User:M1ss1ontomars2k4|M1ss1ontomars2k4]] ([[User talk:M1ss1ontomars2k4|talk]]) 03:56, 29 September 2008 (UTC)
:I'm not sure if this is an appropriate question for the science reference desk (it being about Matlab and being homework), but I'll try to help a bit. First of all, I wonder if you haven't misinterpreted the assignment - the function polyval(p,x) evaluates a polynomial at x that has the values p(n) as its nth coefficient. I'm not sure what you mean when you want to "convert an array into an anonymous function". I guess what I'm saying is...the anonymous function you have described certainly does something, but it's not clear to me why you don't like it. Could you be more specific about what you want the anonymous function to do? --[[User:Bmk|Bmk]] ([[User talk:Bmk|talk]]) 04:58, 29 September 2008 (UTC)
:I'm not sure if this is an appropriate question for the science reference desk (it being about Matlab and being homework), but I'll try to help a bit. First of all, I wonder if you haven't misinterpreted the assignment - the function polyval(p,x) evaluates a polynomial at x that has the values p(n) as its nth coefficient. I'm not sure what you mean when you say you want to "convert an array into an anonymous function". I guess what I'm saying is...the anonymous function you have described certainly does something, but it's not clear to me why you don't like it. Could you be more specific about what you want the anonymous function to do? --[[User:Bmk|Bmk]] ([[User talk:Bmk|talk]]) 04:58, 29 September 2008 (UTC)

Revision as of 05:00, 29 September 2008

Welcome to the science section
of the Wikipedia reference desk.
Select a section:
Want a faster answer?

Main page: Help searching Wikipedia

   

How can I get my question answered?

  • Select the section of the desk that best fits the general topic of your question (see the navigation column to the right).
  • Post your question to only one section, providing a short header that gives the topic of your question.
  • Type '~~~~' (that is, four tilde characters) at the end – this signs and dates your contribution so we know who wrote what and when.
  • Don't post personal contact information – it will be removed. Any answers will be provided here.
  • Please be as specific as possible, and include all relevant context – the usefulness of answers may depend on the context.
  • Note:
    • We don't answer (and may remove) questions that require medical diagnosis or legal advice.
    • We don't answer requests for opinions, predictions or debate.
    • We don't do your homework for you, though we'll help you past the stuck point.
    • We don't conduct original research or provide a free source of ideas, but we'll help you find information you need.



How do I answer a question?

Main page: Wikipedia:Reference desk/Guidelines

  • The best answers address the question directly, and back up facts with wikilinks and links to sources. Do not edit others' comments and do not give any medical or legal advice.
See also:


September 23

Mercury's core

Is Mercury's core hot or cold. If hot how hot is it. Over 5000 egrees C?--57Freeways 00:17, 23 September 2008 (UTC)[reply]

It's certainly not cold and not over 5000 centigrade (or over 9000). While I couldn't find an exact number, Mercury's core is primarily iron and recent data suggest that it's molten iron, which puts it somewhere between 1538 and 2862 centrigrade. -- MacAddct1984 (talk • contribs) 02:30, 23 September 2008 (UTC)[reply]
Around 1850 C. [1] Dragons flight (talk) 04:19, 23 September 2008 (UTC)[reply]

pet birds

which birds when kept as pets must be fed haraam foods in order to survive? I am thinking about buying a parakeet of some species but I do not wish to purchase a bird which requires this. —Preceding unsigned comment added by 90.240.33.166 (talk) 00:34, 23 September 2008 (UTC)[reply]

Haraam ("forbidden") foods being basically pork, alcohol and other kinds of meat that are not appropriately slaughtered? I think that almost every species of bird can be fed on a diet that would not violate the Islaamic dietary rules. Parrots eat seeds and nuts and stuff like that. SteveBaker (talk) 01:21, 23 September 2008 (UTC)[reply]
Vultures? -- MacAddct1984 (talk • contribs) 02:32, 23 September 2008 (UTC)[reply]
You could keep a vulture on (say) lambs killed and prepared in haraam-compatible ways. SteveBaker (talk) 09:46, 23 September 2008 (UTC)[reply]
Insectivorous species might be tricky. Or Halal article states that 'all insects except for the locust' are considered haraam. Might be interesting to find out if the Islamic definition of 'insect' matches the scientific one - if grubs or spiders would also be haraam, for example... --Kurt Shaped Box (talk) 15:45, 23 September 2008 (UTC)[reply]
The most common pet parakeet is the Australian budgie. They can survive entirely on store-bought bird-seed and the occasional cuttlefish bone. APL (talk) 02:51, 23 September 2008 (UTC)[reply]
Hmmm.... I see that the article contradicts me. However, my budgie has been surviving for 11 years on nothing but store bought "parakeet seed" so I assume store bought parakeet seeds contain a mix of seeds to ensure proper dietary balance. Regardless, it's still just all seeds. APL (talk) 02:56, 23 September 2008 (UTC)[reply]
Incidentally, Cuttlefish are mollusks. If that's not acceptable, you might want to check a pet store for artificial calcium supplements for birds.APL (talk) 03:35, 23 September 2008 (UTC)[reply]
Indeed, budgies/parakeets (While budgerigar, or just budgie for short, is the correct term, "parakeet" is practically synonymous with it even though it technically refers to a broader group of parrots) can survive on seeds, however fruits and store-bought pellet diets are considered much healthier. Incidentally, I don't know much about Islamic customs, but from the wikipedia article on Islamic dietary laws it seems that most of the restrictions are about meat products. Since most parrots are herbivores, I doubt you'll run into much problems getting your bird to conform to Islamic dietary restrictions. --Sapphire Flame (talk) 12:28, 23 September 2008 (UTC)[reply]
Store-bought budgie seed is often vitamin/mineral enriched. One brand of seed in the UK contains special 'coated seeds' (which my indoor bird usually avoids). There is debate in the bird-keeping community when it comes to pellets vs. seeds. Some consider the advice from (some) veterinarians to feed pellets to be paid endorsements on behalf of the pet food companies. Other vets recommend seed, as part of a balanced diet containing fruit and veg, with access to cuttlebone and a mineral block. Try giving your bird a slice of apple or raw carrot - those are big favourites amongst my budgies. You might have to let the bird see you eating the same in front of him several times before he'll go near it though. --Kurt Shaped Box (talk) 16:03, 23 September 2008 (UTC)[reply]

quantifying vitamin C's preservative properties

OK, I need some NUMBERS on Vitamin C's ability to prevent food decomposition, compared with other preservatives, but for some reason all the sources online (even Google scholar!) seem fine with reporting the idea that Vitamin C is somehow a preservative, without any numbers whatsoever. This is so frustrating! I see no comparison of Vitamin C's preservative ability with other preservatives, concentrations of Vitamin C injected into food plotted against rotting time, etc. (or something like that) ... where can I get this information? 199.111.200.70 (talk) 01:32, 23 September 2008 (UTC)[reply]

Maybe a refined search for ascorbic acid would be more helpful? Cross-referencing ascorbic acid and preservative seems to turn out some good results [2] -- MacAddct1984 (talk • contribs) 02:41, 23 September 2008 (UTC)[reply]

Why are some stars bright?

why are some stars bright —Preceding unsigned comment added by 117.193.164.137 (talk) 02:04, 23 September 2008 (UTC)[reply]

Either because they are bigger than others or because they are closer than others. See apparent magnitude for the maths. Plasticup T/C 03:21, 23 September 2008 (UTC)[reply]
It's due to a combination of luminosity (how much light the star emits) and distance from the Earth. The size of the star is indirectly related; large stars tend to emit more light. Axl ¤ [Talk] 09:28, 23 September 2008 (UTC)[reply]
Also, if you're seeing a really bright star that's much brighter than every other star in the sky, chances are it's actually Venus. --Sapphire Flame (talk) 13:25, 25 September 2008 (UTC)[reply]

Planets interior

Is Mars core hot for certain. They said it's sulfur not iron. If hot is Mars as hot as our core. What about Pluto. Is it's core hot or cold. Scientist found out it is made of ice silicates of it's center.--57Freeways 02:54, 23 September 2008 (UTC)[reply]

All planetary interiors will be hotter than their exteriors because cooling occurs primarily at the exterior. Dragons flight (talk) 04:21, 23 September 2008 (UTC)[reply]
Is that so? What about planets that have reached thermal equilibrium? There are some exoplanets that are quite old - PSR B1620-26 b is supposedly 12.6 billion years old. On what timescales do planets cool down? --Stephan Schulz (talk) 07:47, 23 September 2008 (UTC)[reply]
For the Earth, the interior temperature drops something like 100 K per Gyr from a core temperature of ~6000 K. Gas giants have a much bigger mass to cool, and a much higher initial temperature, but they also have the benefit from more efficient convention. Even so, Jupiter currently has an estimated core temperature of ~25,000-35,000 K at an age of 4.5 Gyr. In all likelihood your very old planet still has a very warm core. Dragons flight (talk) 08:19, 23 September 2008 (UTC)[reply]
Ok. So in hypothetical case of a planet in thermal equilibrium... ;-) --Stephan Schulz (talk) 08:38, 23 September 2008 (UTC)[reply]
In your hypothetical case, the core temperature is equal to the surface temperature by definition (thermal equilibrium). Axl ¤ [Talk] 09:35, 23 September 2008 (UTC)[reply]
Not necessarily - thermal equilibrium just means that core and surface temperatures are not changing over time. They do not have to be equal if there is a source of heat energy such as those mentioned by Coneslayer below. Gandalf61 (talk) 13:18, 23 September 2008 (UTC)[reply]
Also, it's an oversimplification to imagine a planet as an inert body that's cooling off from some initial temperature. Jupiter is still converting gravitational potential energy to heat (by shrinking), and the earth's interior is heated through radioactive decay. Bodies can also be tidally heated if they orbit close to a massive body. -- Coneslayer (talk) 11:35, 23 September 2008 (UTC)[reply]
This depends on the size of the planets. For Pluto, geenral answer is nobody knows, but one guess is warm enough for water to evaporate but unlikely to be over 500 C or 1000 C. For Mars it is made of iron oxide, so it's core must be hotter than Mercury's core but unlikely to be hotter than ours. The good range of Mars core is 2000-4500 Celsius. Venus' core orange-yellow glow must be same temp. as our core perhaps little cooler. All gas giants core si definitely hotter than surface of sun. It's mantle must be so white as a dense vapor fluid the rockets burn. Even Neptune's core is close to 20,000 C.--57Freeways 23:23, 23 September 2008 (UTC)[reply]

chromatography

If I made a substance that was supposed to be asprin how could I use chromatography to tell if it is really asprin? —Preceding unsigned comment added by 209.97.86.194 (talk) 03:03, 23 September 2008 (UTC)[reply]

You can try looking at Chromatography - in particular note that there are many different kinds of chromatography so the specific method will vary. The very rough answer is that you perform your chromatography of unknown-but-suspected-to-be-aspirin substance and compare the results to those of a known-to-be-aspirin substance that undergoes the same test. This may involve getting some aspiring and doing the test yourself, or referring to some kind of standard set of results. Confusing Manifestation(Say hi!) 05:18, 23 September 2008 (UTC)[reply]

How literate in English would a person have to be in order to practice neosurgery in the USA?

I came here from a political forum on which a user (who is basically a troll) claims that she is a neurosurgeon but is barely literate in English. Here is an example post by her:

"For you are the putrid c-nt who no one like and you threads are the stupid and the boring like you."

What I want to know is if it is even possible to have this level of literacy in the English language and be a neurosurgeon in the USA. If one can barely even construct a basic sentence, how could one possibly understand all the complicated terms used in that profession? This claim seems ridiculous to me. Another thing which arouses my suspicion is that this user has on multiple occasions invited other members of the forum to come and watch her perform surgeries. Would guests even be allowed to watch doctors perform surgeries in the USA? In addition to these almost certainly bogus claims, this user has claimed to be a supermodel of Indian origin and has posted pictures of an attractive woman which she claimed to be her own, but a quick google search revealed them to be pictures of Aishwarya Rai.

There is also another user on the forum who is basically a troll who claims to know this user personally and defends her bogus claims, which makes me suspect sockpuppetry. I just want to know if her claims about being a surgeon are even the least bit credible, given her level of literacy.--68.93.135.252 (talk) 05:15, 23 September 2008 (UTC)[reply]

Why do you even ask when you know the answer? Do you want proof, to stand her up? Why are you feeding the troll? Why hasn't she been banned? Avnas Ishtaroth drop me a line 05:25, 23 September 2008 (UTC)[reply]
It's basically an unmoderated political forum. Pretty much the only thing that will get you banned is posting child porn.--68.93.135.252 (talk) 05:29, 23 September 2008 (UTC)[reply]
Taking the question at face value – I've done medical transcription, and had occasion to listen to medical residents from India, China and Vietnam among other places; some of them could be very hard to understand, but I can't imagine any of them saying "the [adjective]". —Tamfang (talk) 07:41, 23 September 2008 (UTC)[reply]
I don't think this snippet of text constitutes proof at all. Some fairly smart people just don't bother to type too carefully or check grammar/spelling in forum-type posts. I agree though that the claims of being a supermodel and posting of fake photos is not a particularly positive sign. However, it's all circumstantial evidence - the way neurosurgeons behave in their off hours could easily be unrelated to how they behave in their work. Sock puppets are often easy to nail because they all make the same grammatical/spelling errors. Look for a word that this person always misspells and see if you can subtly get one of the socks to use it several times in a post. If you see the same misspelling - then you have some evidence. If you can find several different words like that - then you have quite strong evidence that they are in fact the same person. But in an unmoderated free-for-all, you have very little recourse. I recommend simply ignoring this person's posts - reply to everyone else directly but simply never mention or refer to this one person's posts. SteveBaker (talk) 09:41, 23 September 2008 (UTC)[reply]
Honestly, who cares? On the internet you are whatever you want to claim to be. And not a single person has to believe you. --98.217.8.46 (talk) 13:22, 23 September 2008 (UTC)[reply]
Indeed. -- Coneslayer (talk) 13:26, 23 September 2008 (UTC)[reply]
Indeedly-doodly. AlmostReadytoFly (talk) 14:55, 23 September 2008 (UTC)[reply]
Many doctors in the U.S. got their medical training in non-English speaking countries and barely speak English. They might write and speak more fluently and with greater literacy in their native language. I believe that they have to pass the "Test of English as a Foreign Language" to practice medicine here, but many have failed it multiple times, and maybe one day they got lucky and scored higher than their actual language proficiency would dictate, or maybe they cheated (getting the answers in advance via corrupt proctors,or from a n administration earlier in a different time zone, hiring a substitute, electronic communications). A magazine article a few years ago complained about this lack of English proficiency, in which an Asian doctor in a US hospital asked the patient "Who fam doc?" repeatedly until a bilingual nurse told them he wanted to know the name of their family doctor. So in my view, yes, there could be a practicing surgeon in the U.S who might write the sample posting. Poor grammar, poor spelling, limited English and a lack of attention to correct typos as well as a generally bad attitude could yield the sample document. Edison (talk) 15:31, 23 September 2008 (UTC)[reply]
Out of curiousity, do you have a link to this person's posts? —Preceding unsigned comment added by Schwarzes Nacht (talkcontribs) 01:30, 25 September 2008 (UTC)[reply]
Most medical doctors wishing to practise in Great Britain, and coming from outside the European Union, are required to demonstrate proof of their English aptitude (in all four skills -- reading, writing,listening, speaking). This is normally done by taking the Academic module of the IELTS exam. Not sure whether the US has a specific equivalent. BrainyBabe (talk) 11:53, 27 September 2008 (UTC)[reply]
In the US, it would be the TOEFL. ArakunemTalk 11:59, 27 September 2008 (UTC)[reply]

Hydrogen ballooning

Does anyone still use hydrogen for manned balloons, and if so, what do they use for balloon fabric and where can I get some? --67.185.172.158 (talk) 05:53, 23 September 2008 (UTC)[reply]

According to our article on balloons, hydrogen is "not widely used for aircraft since the Hindenburg disaster because of high flammability (except for some sport balloons as well as nearly all unmanned scientific and weather balloons)." As for the fabric, no clue. -- Captain Disdain (talk) 09:25, 23 September 2008 (UTC)[reply]
I had to research something like this once for a school project, and eventually found a reference (i don't remember where) to ruberized silk. Didn't know then and don't know now what that actually is though 124.243.155.3 (talk) 11:21, 23 September 2008 (UTC)[reply]
For the Roziere it might be dacron or/with ripstop nylon. Mion (talk) 12:20, 23 September 2008 (UTC)[reply]

the Martian sky

If you stand on the equator on Mars at midnight at the northward equinox, what constellation is directly overhead? In other words, where does the Sun-Mars vector point at Ls=0? —Tamfang (talk) 07:43, 23 September 2008 (UTC)[reply]

According to this Martian calendar, it is probably Sagittarius (constellation). You've probably already found Timekeeping on Mars, that article has a lot of information. Nimur (talk) 22:26, 23 September 2008 (UTC)[reply]

Looking for specific puzzles

I found via wikipedia maybe a year ago a page containing possibly several hundred logic puzzle images that had first been designed (from memory) around the turn of the previous century. The wikipedia page itself was brief, what I'm really after is the page it linked to.

The external page had links to at least 130 pairs of images on the site. They were colour-coded on this page, to indicate who created each puzzle, with the names and references at the bottom of the index page. The puzzles themselves were two images, and you had to determin what the unique difference between them was. Each subsequent pair of images had a different answer. Does this sound familiar to anyone? Thanks in advance. —Preceding unsigned comment added by 219.89.54.45 (talk) 10:38, 23 September 2008 (UTC)[reply]

accelerating charged particles and radiations

Why do accelerating charged particles emit radiations? —Preceding unsigned comment added by 210.212.187.69 (talk) 14:13, 23 September 2008 (UTC)[reply]

I think that particles only emit radiation when they collide with other matter (assuming the particles were non-radioactive to begin with), because their kinetic energy is turned into radiation due to the conservation of energy. This is true for anything with mass moving at extremely high relative velocities. ScienceApe (talk) 19:59, 26 September 2008 (UTC)[reply]
You would want to look at bremsstrahlung and the question archives, as we touched on a similar question a few weeks ago. :-) EverGreg (talk) 21:40, 27 September 2008 (UTC)[reply]

Help from more experienced molecular biologists: statistical software packages?

I've been tasked with finding a statistical software package to help us with some of the more routine statistical analysis in our molecular biology lab (mostly ROC curve and multivariate analysis), and I was looking for recommendations from the community. Most of the information I'm finding is from company web sites that (naturally) insist their package will change your life, or from other investigators who really only have experience with a single package, if any at all. Is there anybody out there who some experience in this subject who can recommend anything that stands out and is worth the expense? Many thanks! – ClockworkSoul 15:23, 23 September 2008 (UTC)[reply]

Note: I just found the list of statistical packages, but some practical advice would be invaluable! – ClockworkSoul 15:26, 23 September 2008 (UTC)[reply]
I don't know about their abilities with the specific methods that you mention, but here's what I do know about a few different packages. SAS has the reputation for being high powered and versatile, but with a steep learning curve and ridiculous amounts of documentation to wade through. I don't know if they make a version of SAS with a GUI yet, but for a long time it was all command line driven. The more user friendly-packages I've used include SPSS, Stata and Minitab. I found it easy to write scripts for complex and repetitive data manipulation and analysis in Stata, but I never attempted scripting with the other packages for comparison. I don't have much else intelligent to say regarding a comparison of these three except that Minitab seems to have a big user-community, which is always helpful. Since they probably all now perform most of the same methods, you might find that they differ more in terms of the prettiness of the charts they can produce. ike9898 (talk) 17:25, 23 September 2008 (UTC)[reply]
In the bioinformatics community, which is close to your line of work, R (programming language) is an important tool. The Bioconductor package is built on R. The learning curve for R is somewhat steep, but it pays off. There may be a bioinformatics user-group somewhere nearby that you could get in touch with, and they might organize courses. R is an excellent tool for multivariate analysis, and also excels at making pretty graphics. I have no experience in using R for ROC curves, but a google search suggests that packages are available for the task. --NorwegianBlue talk 20:56, 23 September 2008 (UTC)[reply]
SAS does have a GUI now, called SAS Enterprise Guide. It's basically a front-end that calls an installation of SAS either locally or on a server. It's probably a bit easier than using SAS directly (which itself has a kind of GUI/text editor interface to the command line interpreter), but to do anything really powerful statistically you still need to be able to wade into the code a bit. Confusing Manifestation(Say hi!) 22:55, 23 September 2008 (UTC)[reply]

project

Yes, I work for a company that builds the fuel pump for some cars. We've had a lot of problems with scrap- (meaning problems with the armatures.) I mean a lot of bad armatures. I need someone to help me with some suggestions is to how some of this could be prevented. I hope too, that this is a question that is suitable. Thanks, tqr —Preceding unsigned comment added by Quantinatqr (talkcontribs) 15:40, 23 September 2008 (UTC)[reply]

Electrical armatures like all components will eventually break down. I don't know of any sure ways to prevent this, and I guess that's why armature-winders are employed to locate broken coils and to repair or replace them with new ones, or completely rewind all the coils from scratch. Perhaps a more dense iron core and better quality coils would help, although I'm sure the company you work for will have researched these possibilities already. JessicaThunderbolt 16:18, 23 September 2008 (UTC)[reply]
Let that be a lesson to you: NEVER work with armateures. Instead, use professionals, then you wont have any troubles. —Preceding unsigned comment added by 79.76.251.108 (talk) 22:57, 23 September 2008 (UTC)[reply]

Your starting point should be to identify the problem a little more specifically. Why are they breaking? Can you fix that problem. Plasticup T/C 04:42, 24 September 2008 (UTC)[reply]

One reason for a lot of older fuel pumps failing is the introduction of ethanol into our fuel supplies. Ethanol conducts electricity where gasoline does not - some older fuel pumps had exposed metal parts that were at 12 volts - which now have a low-resistance path to ground resulting in overheating and reduced pump pressures. Ethanol also chews it's way through rubber seals - modern fuel pumps should have synthetic plastic seals everywhere. But I would imagine that both of these issues are very well known facts for the designers of fuel pumps and it's hard to imagine a well-designed modern pump having this problem. The problems I've seen have been with fuel pumps on old cars - more than 15 years old.
Another possibility is that fuel pumps that are immersed inside the gas tank to keep them cool are at risk of overheating when people run their cars very low on fuel. I suppose that it could be that the increased cost of gasoline over the past year or two is somehow causing more people to leave it to the very last minute before filling up their tanks - thereby causing the fuel pump to overheat more - and for longer periods than it used to.
Failing that - you have to find an armature manufactured in the era before they all started failing and compare it VERY carefully with the more recent ones - literally put it under the microscope. Maybe they changed the formulation of insulation on the windings? Maybe they use a different gauge of wire or a different number of turns? Maybe the copper in the wire comes from a different place - maybe it's not so pure perhaps? Perhaps a different adhesive is being used somewhere and it's mildly corrosive? Something must have changed - and if it's not the environment the things are running in - then it must be the design or the construction.
SteveBaker (talk) 01:53, 25 September 2008 (UTC)[reply]

object at infinity

if mirror is concave and object is at infinity ,is there any measurement of infinity,i-e what it should be ,greater then 3f,4f ,5f or others —Preceding unsigned comment added by Khubab (talkcontribs) 15:53, 23 September 2008 (UTC)[reply]

It appears to me that this is asking about the angle of reflection in the concave mirror. The closer an object is to the mirror, the sharper the angle. The further away an object gets, the angle of reflection approaches a straight line. At infinity, since it is impossible to be at infinity, the angle would be a straight line, which is impossible to achieve. -- kainaw 17:15, 23 September 2008 (UTC)[reply]
I think the OP is probably asking how far away an object should be in order to be treated as being "at infinity" for practical purposes. This distance can be expressed as a multiple of the focal length (hence 3f, etc.). I think the rule-of-thumb that I learned was that 200f is "infinity" for most purposes, but I am not a reliable source. -- Coneslayer (talk) 17:36, 23 September 2008 (UTC)[reply]

evidence of increase in iron content in well water

What could cause this increase in iron content from a scale of a 1 to a 7 in one year? —Preceding unsigned comment added by 69.250.196.171 (talk) 17:09, 23 September 2008 (UTC)[reply]

The water could become more acid, or anoxic conditions could be reducing insoluble ferric compounds to mobile ferrous ions. Iron is a natural component of ground water, and it may vary in concentration when it comes from different aquifers - exposed to different rocks. Graeme Bartlett (talk) 21:18, 23 September 2008 (UTC)[reply]

Actually making money from a patent

I'm looking for information about how often a patent holder actually makes some money from his patent, either by licensing it, using it himself, or some other means. More precisely, I'm interested in the percentage of patents that never benefit their holder in any concrete way. ike9898 (talk) 17:11, 23 September 2008 (UTC)[reply]

You don't make money from the patent itself, you make money from whatever it is you've patented. The patent just stops anyone else using it and stealing your market. To make money from an invention you need to make whatever it is you've invented and sell it (or license someone else to do so). --Tango (talk) 17:33, 23 September 2008 (UTC)[reply]
I think ike9898 understands that... he's asking how often that actually happens, versus a patent sitting around collecting dust (neither being licensed nor put into production by the inventor). -- Coneslayer (talk) 17:38, 23 September 2008 (UTC)[reply]
But do you wish to include blocking patents (well darn, we have no article on them) which intentionally collect dust. Someguy1221 (talk) 18:14, 23 September 2008 (UTC)[reply]
It's an interesting question. I'd guess offhand that the vast majority of patents never make a cent directly or indirectly. But it would be really interesting to follow up on, at least with a study of, say, 1000 patents chosen at random. There are many different ways that a patent can be "of value"—most of which not including that they mean you can sell the product at the market (e.g. defensive patenting is of very high value even though it has no direct revenue, it just protects against hypothetical revenue depletions). --140.247.253.44 (talk) 19:35, 23 September 2008 (UTC)[reply]
The average cost of obtaining a patent is ~$30,000, so I'd imagine nearly everyone goes in with the expectation that it will either make them money or prevent them from losing money (whether directly or indirectly). What fraction of patents actually realize those revenues? I have no idea. It would be interesting data to have though. Dragons flight (talk) 19:58, 23 September 2008 (UTC)[reply]
Many technology corporations/companies will pay the inventor for submitting a patent, and provide additional financial incentives to the inventor at each additional stage of the patent procedure (filing, granting, etc). In this regard, the inventor can earn fixed quantities of money directly from the corporation; the corporate office is then responsible for the additional challenge of monetizing the actual invention. To capitalize on this, you should first seek employment in the research wing of a company with some such inventor compensation plan. Nimur (talk) 22:14, 23 September 2008 (UTC)[reply]
Well, keep in mind that most "inventors" are really corporations anyway. The age of the small-time inventor (in the US anyway) died a long time ago. As to whether capitalizing on an invention in a corporation—it really, really depends on the corporation's patent plans. --98.217.8.46 (talk) 22:33, 23 September 2008 (UTC)[reply]
Where are you getting that figure from? That sounds awfully high. I'm assuming you're talking about lawyer's fees in particular, as the patent office fees are not very steep. Are you referring to individual inventors or corporate inventors? --98.217.8.46 (talk) 22:33, 23 September 2008 (UTC)[reply]
Our article on patent, for example, says that in 2005 the average cost of filing and maintainence during first 10 years in Europe was ~32,000 Euros. Most of the cost is patent attorney costs. $10-15 thousand to prepare the filing, and roughly that much again to repsond to challenges and finish the process during the next few years would not be uncommon. Of course the complexity of the invention matters, and I'm sure one can get patents on the cheap, but you won't necessarily save in the long run by doing so. Dragons flight (talk) 22:56, 23 September 2008 (UTC)[reply]
Contingent fee trial lawyers. You file a patent for something you think will be invented anyway, and then you wait for microsoft or yahoo to infringe, then you fire the trap. The best example of this is Research in Motion. Additionally, many companies offer "advanced settlements" so that not only do you have zero-risk by accepting a contingency fee deal, you can sign a contract with a laywer at the same time you receive an advanced settlement (which you don't have to repay unless you win). This is risk-free profit, and its like those people who register domain names, oneday hoping the company will buy it from you for $1M. However, I follow this, and the patent system is going through major overhaul and is trying to blend in many experts including engineers and private sector MBA's and thus squeezing out the huge percentage of lawyers and bureaucrats involved in the process. Sentriclecub (talk) 03:28, 24 September 2008 (UTC)[reply]

Cause of death

Not a medical question in the sense that if affects anyone that I know, is purely hypothetical. There is a cause of death which presents with hugely bloated stomach and fecal vomiting (post mortem, i.e. fecal drainage from the mouth) I'd imagine it is some kind of intestinal/gastro bleed or leak? SGGH speak! 18:21, 23 September 2008 (UTC)[reply]

I hope this isn't a request for medical advice. If so, you're a little late :-) Chances are it could be a result of a number of things. Gastrointestinal perforation is the first to spring to mind. Also, perhaps, though less likely, being upper GI, is Boerhaave syndrome. Fribbler (talk) 21:54, 23 September 2008 (UTC)[reply]
Various types of bowel obstruction can cause faecal vomiting, if severe enough. Many of these won't cause death in and of themselves, but aspiration of the vomit could do so. In one imaginary scenario (since we are discussing such, I believe?), the individual might lose consciousness through the pain and dehydration caused by the obstruction, and when retroperistalsis causes the backlogged bowel/stomach matter to be vomited, some leaks through the mouth, and some finds its way into trachea and lungs, causing, effectively, death by drowning. That do you? Gwinva (talk) 22:24, 23 September 2008 (UTC)[reply]

It's hypothetical in that I do not know anyone who has died of this, but I am aware that this manner of death does occur and was wondering what caused it. The perforation was my guess. Unfortunately searching fecal vomit does not turn up too much. Thanks guys SGGH speak! 22:54, 23 September 2008 (UTC)[reply]

True "fecal vomiting" (i.e. vomiting feces) is rare and is usually due to gastrocolic fistula (an abnormal connection between stomach and colon). Feculent vomiting (foul-smelling dark brown vomitus) is a little more common (though still uncommon) and is due to large bowel obstruction. This should be distinguished from brown vomitus that is not foul-smelling. Another commonly used (and abused) term is "coffee-ground vomiting". This is supposed to describe thick black vomitus due to upper gastrointestinal bleeding. Unfortunately the term is misused by junior doctors, nurses and other health professionals; it is incorrectly used to describe brown vomiting. The symptoms of "hugely bloated stomach [abdomen] with vomiting" sounds most likely to be due to perforation. If a post mortem report genuinely states "fecal drainage from the mouth", I would hope that the pathologist also looked closely for evidence of fistula. Boerhaave syndrome can follow vomiting, but is rarely associated with hugely bloated abdomen. Axl ¤ [Talk] 19:49, 24 September 2008 (UTC)[reply]

Mysterious Hamster Illness

Before I begin, I will say that this is not a request for medical advice, as the hamster in question is dead. The question is about what killed her.

I had a hamster (Pearl Dwarf Hamster) and I first noticed she was a little odd when she would give off screams at random, usually when 'sleeping' under all the wood chippings we used for bedding. One night, my wife turned to me and said she was not moving, so I picked her up and found she was dead. He eyes were tightly shut and were caked with blood. Does anyone have any idea what could have caused this? I doubt it was an injury, because we had another hamster in a cage underneath and she had no problems. Please note, they could not have been fighting, as they had no possible contact with each other.

My theory was that it was a brain haemorrhage, but would this cause the eyes to bleed?--ChokinBako (talk) 19:18, 23 September 2008 (UTC)[reply]

Allergies can cause this, or porphyrin staining if it is albino, or a viral infection. Allergies might have been from the bedding, possibly some sort of dust, or something like that. You have my sympathies, it isn't nice to loose a pet! SGGH speak! 19:31, 23 September 2008 (UTC)[reply]
Out of curiosity, what sort of wood chips were you using for the bedding? I used to work at a pet shop and might be able to give you some answers. Allergies are a big possibility. Also, some hamsters you get from less reputable pet shops will have neurological issues from interbreeding. Believe me, I've seen it all. Red Gown (talk) 21:32, 23 September 2008 (UTC)[reply]

On the more traditional side, any kind of tumor on/in/near the eyes could have done it. Plasticup T/C 01:27, 24 September 2008 (UTC)[reply]

How old was your hamster? The Perl White ones only live 18 months to 2 years - perhaps it was just old age? SteveBaker (talk) 01:34, 25 September 2008 (UTC)[reply]

Ubiquitination: specific E1 needed for particular E2?

I know that a relationship exists in terms of which E3s associate with which E2s but how many E1 ubiquitin activating enzymes are there? Can any E2 gain the ubiquitin protein from any E1? ----Seans Potato Business 19:57, 23 September 2008 (UTC)[reply]

What will happen exactly if all the arctic ice melts?

My 5 year old son is asking me and I want to explain to him exactly what will happen and how this will affect him. Please explain to me as if you were to tell a five year old child. --Anilmanohar (talk) 20:31, 23 September 2008 (UTC)[reply]

Not really a 'simple answer' kind of question. There are dozens of different things which would happen. Polar bears and various seal populations which live on the ice part of the year would lose their habitat and decrease in numbers. Dark sea water would absorb more heat from sunlight than ice would - thereby resulting in increased ocean temperatures. Air above the oceans would also be correspondingly warmer, resulting in weather changes for surrounding land masses and further melting of the 'permafrost'. Sea ice also provides a buttressing effect preventing land ice from falling into the ocean - without it glaciers on Greenland and other arctic land masses would decrease more quickly, resulting in increased sea level rise. Methane and carbon deposits trapped beneath the ice would be released - increasing the concentration of greenhouse gases in the atmosphere and further accelerating global warming. Ships could sail through more often. Oil and natural gas which are currently unreachable through the ice might be extracted. Et cetera. Those are most of the 'big ticket' items, but it would contribute to countless already ongoing changes the world over (species migration northwards and to higher altitudes, glacial retreat, sea level rise, average temperature increases, et cetera). A common misperception is that melting Arctic ocean ice would directly raise sea levels. However, since that ice is floating it is already displacing an amount of water equal to the volume it would melt into. --CBD 21:03, 23 September 2008 (UTC)[reply]
One little experiment you might like to do with your son, if he's interested in a bit of fun home science, will show how melting the Arctic ice cap won't directly raise the sea levels, but melting the Antarctic ice, or the ice on Greenland, Canada, far north Europe, etc will. Take a glass of water and drop an ice cube or two into it. Note that some of the floating ice sticks out above the water (like an iceberg). Mark the water level with a piece of tape, and then wait for the ice to melt. The water level will be the same afterward, even though the ice that was sticking out the top has turned into water.
Then try the experiment again, but this time, hold the ice cube completely out of the water (you could perhaps balance it on a spoon above the glass so as it melts the water runs into the glass). This time the water level will have risen. The floating ice cube imitates the Arctic ice cap, which simply floats above the ocean. The suspended ice cube imitates the ice on Greenland, Antarctica, etc, that is, ice sitting on top of continents, held out of the water. Of course, we can't selectively choose which ice melts if the globe warms up, but this will be an interesting and easy way to see that it isn't the melting Arctic ice cap that will result in raised sea levels, it's all the other ice. Maelin (Talk | Contribs) 01:07, 24 September 2008 (UTC)[reply]
Actually, when ice that's floating on the sea melts, it does contribute a little bit to sea level rise, although not nearly as much as it would if the ice had been resting on a land mass. The reason is because (unfrozen) fresh water, which the ice is a frozen version of, is less dense than the salt water of the sea. See [3]. Red Act (talk) 05:13, 24 September 2008 (UTC)[reply]
Yeah - exactly. The ARCTIC ice melting doesn't DIRECTLY do much to the sea level at all because it's all floating ice. However, if the ANTARCTIC ice melts (well - if MORE of it melts - lots of it is already gone) - then the sea level will rise by over 60 meters(!). It the snow and ice on Greenland melted, that would raise the ocean levels by a further 7 meters. Additionally, because water expands as it's heated, there would be further increases in sea level even after all the ice had melted. When you add up all of that - including glaciers, mountain-top ice, etc - you get a total sea level increase of perhaps 100 meters. That most extreme situation would leave almost all of the world's major cities under water. Some of the effect are tough to predict because the weight of all of that water presses down on the continents - as the weight of the antarctic ice was released, the antarctic continent would rise up - which would displace yet more sea water...the pressure of that extra water flowing inland onto the other continents would press them down - causing yet more flooding. Even more depressing is that the ice is bright white and reflects most of the sunlight that hits it off into space. When the ice melts, it's replaced by darker land or much darker ocean which absorbs more sunlight and accellerates global warming to an even greater extent. If the ocean temperatures rise enough then deep ocean deposits of frozen methane would melt - releasing enormous amounts of the stuff into the atmosphere. Methane is a much worse greenhouse gas than Carbon Dioxide - so we'd be even more screwed up. As the earth heats up - more water evaporates from the oceans. If this condenses out as cloud, then it reflects sunlight away and that would help to slow the heating due to sunlight...but sadly, water vapour is ANOTHER nasty greenhouse gas...so there is no saving us at that point.
The worst of this is that it's like rolling a heavy ball up over a low hill. All the while you're rolling it up the hill, you can stop pushing and it'll gradually roll back down again. But if you push it just a little bit too far - the ground starts to level off - and even a gentle push will keep it going. But once you cross the brow of the hill - the ball starts rolling down the opposite side - and no matter that you stopped pushing, it's going to keep on going until it eventually rolls to a halt far away on the other side. We really don't know for sure where the top of the hill is - but the accelerating rate of arctic and antarctic ice melting suggests that the slope is at least levelling off and we can't be far from that tipping point beyond which we're screwed. There is some evidence that this is exactly what happened to Venus - and if that's where the level ground on the other side of the hill is - we're in deep trouble.
SteveBaker (talk) 01:11, 25 September 2008 (UTC)[reply]
Other human issues so far unmentioned: the lives of the Inuit people and Eskimos (and Sami for that matter), from Greenland to Canada to Russia, will be changed beyond recognition; at first this will be predominantly bad, but soon more positive may appear, especially in places such as Nunavut, where the Inuit have more control over their lives. Migration will increase; northerners will move south to the cities, as the economies become more intertwined; southerners will move north, as possibilities expand. Issues of the North will be more prominent, culturally and politically, in their related polities to the South. (There might even be a former governor of Alaska as President of the United States, for example. At a trivial level, Nanook of the North will be reissued, and hockey will get hotter.) Animal and plant species will change hugely, from the migration patterns of caribou and the great rorquals, to an ocean-wide bloom of phytoplankton, which entrepreneurs will attempt to harvest as a human food source. Mineral and oil exploration will become easier and cheaper, relative to now, although not necessarily relative to other places. The Northwest Passage will become a year-long reality and Canada will attempt to back up its claim to the Arctic Ocean as territorial waters, with a predictable response from the United States. Tourism to the North Pole will pick up, as it has to Antartica, Mount Everest, and space. Santa Claus will travel by flying whale rather than reindeer. At that point, my crystal ball grows cloudy. BrainyBabe (talk) 12:35, 27 September 2008 (UTC)[reply]

Saturn's disc color

Is Saturn truly blue-silver in disc color almost like Uranus or Neptune. When I went to 6th grade science camp on one night we had astronomy lesson the scientist told me Saturn's sphere color is blue-silver instead of yellow or tannish color.--57Freeways 22:34, 23 September 2008 (UTC)[reply]

Saturn looks a kind of yellowish-gold in a telescope. Most of the photos in our article show it as a yellowish orange. It does tend to look more blue when you are seeing the disk of the planet through the rings. SteveBaker (talk) 00:40, 25 September 2008 (UTC)[reply]

Tooth extraction

Not seeking med advice -- just curious. If a tooth is broken off at or below the gum line, how does the dentist remove it? (as she cant get hold of anything) I just had it done but wasnt watching at the time.--GreenSpigot (talk) 22:51, 23 September 2008 (UTC)[reply]

I had my wisdom teeth out last month and I think that the root of one of the teeth broke off inside the gum. For the next four weeks little pieces of tooth would (each over the course of several days) force their way to the surface, through it, and out. Presumably my body was forcing it out through the process of granulation. It's not exactly a surgical technique, but it was pretty cool to see my body ridding itself of these shards. Plasticup T/C 01:24, 24 September 2008 (UTC)[reply]
The same thing happened to me when I had one of my molars removed - but with fragments of jawbone. The infection beneath the tooth had started to consume the bone and my dentist had to have a good hack away once the tooth was out, see? --Kurt Shaped Box (talk) 01:32, 24 September 2008 (UTC)[reply]

I really, really wish I hadn't read this. DuncanHill (talk) 01:35, 24 September 2008 (UTC) [reply]

I read about your condition on WebMD and at first I was worried that it could be pieces of jawbone. Inside my mouth these fragments felt huge, but when the first one fell out it was no larger than 1 mm3 and I figured it couldn't be anything too serious. Plasticup T/C 04:37, 24 September 2008 (UTC)[reply]
Also from personal experience, both of my upper canines snapped off at the gum line (curse you refrigerated Chunky Kit Kats) and were removed by cutting away the gum, drilling out the middle of the remaining stump and what seemed to me to be an inordinate amount of sheer pulling force. For my wisdom teeth; the dentist drilled out the centres, placed his pliers/tools/mouth-butchery devices inside and twisted forcefully until the tooth shattered. Yay NHS... Nanonic (talk) 12:03, 24 September 2008 (UTC)[reply]
Also, see Dental extraction#Types of extraction with some lovely pics. Nanonic (talk) 12:08, 24 September 2008 (UTC)[reply]
Thanks all. But I was looking for more detail on the process of extraction. Ive heard that things called Dental elevators are used to lever up the tooth. Unfortunately, no article!--GreenSpigot (talk) 03:17, 25 September 2008 (UTC)[reply]

find the resonance frequency of a tube

In order to do a high school science experiment, I need to find out the resonance frequencies of a certain tube that I'll be using. The trouble is I don't know the exact specifications of the pipe I'm going to get, but I assume there's a formula out there somewhere. I already looked at the Acoustic resonance article, and that had a formula, but it didn't take into account diameter (as well as other variables I might not know about), which I assume is a large part of the equation, as well as possibly material the tube is made out of and the things inside the tube (different types of gasses), although these aren't quantifiable variables.

I can imagine that a seasoned science teacher/student can imagine what experiment a high school senior would want to do with this information, but I think I'll keep that to myself for now. Thanks in advance for the help. —Preceding unsigned comment added by Hypershadow647 (talkcontribs) 22:57, 23 September 2008 (UTC)[reply]

OK Why do you think the dia is not mentioned? —Preceding unsigned comment added by 79.76.251.108 (talk) 23:00, 23 September 2008 (UTC)[reply]
Instead of taking a look at acoustic resonance, try reading the article. It does, in fact, show the formula for air tubes which include correction for the diameter of the tube. -- kainaw 23:09, 23 September 2008 (UTC)[reply]


September 24

Underwater urination

Its easy to pee at sea level, but at what depth below the ocean surface would it become impossible due to the external pressure?--79.76.251.108 (talk) 00:00, 24 September 2008 (UTC)[reply]

This is equivalent to asking if a balloon can leak air at under high external pressure. The external pressure pushes on the outside of the balloon (or your bladder) and forces the contents out. So, no, under any pressure in which a human could survive, urination would not be a problem. -- kainaw 00:10, 24 September 2008 (UTC)[reply]
Agreed, as your question stands, its a simple answer. However, if you were in some sort of vesicle which was at STP (standard temperature and pressure), and you urinated into a system at a different pressure, then plausibly sea water could enter your bladder because of the fluid force being higher on the outside. If your skin was "completely impermeable" then it would be impossible to urinate below a certain depth (and even above this depth, you wouldn't be able to fully empty your bladder, but only an amount less than at STP). Sentriclecub (talk) 03:21, 24 September 2008 (UTC)[reply]
I have never had any problem peeing at any depth below the ocean's surface. I don't think it would matter how deep you go. The pressure in the human body (which consists mostly of incompressible liquid) is equal to the pressure of the surrounding environment. Forcing urine out requires no more effort at depth than at the surface.
Now, as Sintriclcub says, if you were in a submarine with an internal pressure of 1 atmosphere, and you wanted to drill a hole through the hull to pee out of, you would have to compete with the water pressure on the outside to force your urine out. ~Amatulić (talk) 20:10, 24 September 2008 (UTC)[reply]
True, your urine would be much more compact inside your body. It would be just as easy. Open system, free equilibrium. Sentriclecub (talk) 09:58, 25 September 2008 (UTC)[reply]

Death under stress

In a previous question, Kurt Shaped Box said that "If an animal is stressed at the time of death (and strangulation tends to be a stressful experience), various chemicals are released into the bloodstream which (negatively) alter the flavour of the meat and cause it to go off more quickly." Is the same true of humans? Can this be used to determine whether a person died under stress or "peacefully in their sleep"? Is it ever used in court? Plasticup T/C 00:53, 24 September 2008 (UTC)[reply]

A few years ago, scientists used tissue and blood samples from deceased deer to demonstrate that the practice of hunting with hounds was a stressful experience for the quarry (see [4]). So, yes - I'd imagine that there would be discernible signs, for those who knew where/how to look. --Kurt Shaped Box (talk) 01:47, 24 September 2008 (UTC)[reply]
Sadly, I just learned this a week ago, but here it goes anyway. These "various chemicals" are called signals. On the surfaces of cells, there exist various glycoproteins called receptors. Different cells can respond to the exact same signal, in completely different ways! The most cited example is norepinephrine. It will cause different results in different cells because of the physiology of the different tissues comprising massive quantities of similar cells. The main response is the "signals" will cause the vast mega-accumulation of the requested proteins. To answer your question, yes, it could be potentially used in forensic evidence if there are few ambiguities. For example, if a stress hormone causes the same response as a hunger hormone, then if the prosecution argues "look at these protein buildups--the person died in anguish!" the defense would counter "look at these protein buildups--the person died hungry". But yes, as long as there's not much ambiguity, then it could be used as forensic evidence. Remember, the "judge" and "jury" interpret expert testimony. I watched 3 consecutive days of the trial of the accountant who shot up his workplace (famous story, dont know his name) but I think you asked a very good question. It shows that you are creative, and I wish all questions were more like yours that relied on creativity and critical thinking. Its no fun to answer questions that just blue-link to the article, or answer homework questions. Sentriclecub (talk) 03:15, 24 September 2008 (UTC)[reply]
An answer and a compliment. What more could I want? Plasticup T/C 04:32, 24 September 2008 (UTC)[reply]
I'm not so sure. For animals, stress is generally a short-lived matter. The deer gets chased - if it escapes, it's back peacefully nibbling the grass within minutes. We humans can get stressed about (for example) the state of the financial market and remain stressed for days or weeks at a time. Should you happen to "die peacefully in your sleep" during an extreme financial crisis, I think your chemistry might look pretty similar to someone who has been stressed by (say) a burglar. We also deliberately stress ourselves - there are human "adrenaline junkies" who jump out of airplanes, ski down cliffs or drive their cars very fast "just for fun". Those people could be the happiest people in the world and yet show all the signs of high adrenaline in their blood stream. So the chemistry might tell you something about the person at time of death - but because humans are so complicated (behaviorally) - it might be almost impossible to interpret the results in any meaningful or useful way. SteveBaker (talk) 00:25, 25 September 2008 (UTC)[reply]
Again, I'm not talking about emotional stress or some other psychological criteria for definition. I'm talking strictly physiological stress, the kind definitively linked to certain molecules which trigger the vast mega-production of proteins inside the cells of various tissue types. Sentriclecub (talk) 10:00, 25 September 2008 (UTC)[reply]

Human body

We humans are built to withstand atmospheric pressure and we do not expand or contract. But if we were to go high in the atmoshere would our body expand? Also, if we were to go tho the bottom of the ocean, would our body contract?--GreenSpigot (talk) 01:03, 24 September 2008 (UTC)[reply]

Your assumption that humans do not expand or contract is not entirely true. The human body is designed to expand and contract as needed. Just ask anyone who can feel a storm coming as joints (especially knees) react to the change in air pressure. -- kainaw 01:19, 24 September 2008 (UTC)[reply]
I think he means that humans don't blow up like balloons. The small change in pressure inside joints is negligible. The only part of the body that really adjusts to pressure is the inner ear. Youth in Asia (talk) 03:00, 24 September 2008 (UTC)[reply]
Humans do expand and contract. Drive up a mountain with an apparatus attached to your body that disallows your ears to pop, you'll see. But the question you ask is really complicated. Humans are an open system, which means every femtosecond your body adjusts to restore equilibrium. Also, re-think your question in terms of everyday scientific terms. This is a question about volume. At the bottom of the sea, your density would increase. (the water would squish you into a smaller version). You would have the same mass (for simplicity, lets skip the microassumptions), and since your volume decreases, and your mass increases, then viola! your density increases. This is an easy question, but if you try to answer it without science, you may as well discuss evolution vs intelligent design. Hope you find this analogy helpful. Sentriclecub (talk) 03:06, 24 September 2008 (UTC)[reply]
(conflict) Excuse me, Youth in Asia, but the human body absolutely blows up like a balloon. It's called breathing, and it affects the chest, abdomen, really the whole torso. Your abdomen also expands to respond to a large meal. The human body is very flexible in terms of changes in volume. As far as the original question goes, the OP may find our articles on Space exposure, Human_adaptation_to_space#Unprotected_effects, and Uncontrolled_decompression#Fallacies enlightening. In short, if exposed to the vacuum of space you will not explode but you may bloat a bit depending on how long you're exposed. If exposure is < 30s you will probably recover completely. Otherwise you will first lose consciousness, then your blood and bodily fluids will begin to boil, resulting in bloating but not rupture of the body because of the strength and flexibility of the skin (see ebullism). CO2 will be released form the blood, resulting in damaging changes in blood pH, and nitrogen will also evolve from the blood forming embolisms. Ultimately, death will result form hypoxia. --Shaggorama (talk) 06:03, 24 September 2008 (UTC)[reply]
I'm pretty certain, that if you are talking about extraterrestrial space, then the human body expands well beyond the limits of survivabilitity. Yeah, step out of the spaceship, without your suit, and you will instantly "expand" to about 1,000,000 times your original size. I think the relevant article is about the partial pressures of the organic compounds comprising your skin cells, and then everything else will follow. But I'll let you have the last word, I max 10 edits to this page, then I take a 1-month vacation. Sentriclecub (talk) 06:48, 24 September 2008 (UTC)[reply]
Our articles disagree with your expanding hypothesis. --mboverload@ 07:00, 24 September 2008 (UTC)[reply]
The difference caused by stepping out of a spacecraft is the same as it caused by rapidly ascending through 10 metres of water. If you try and hold your breath, your lungs might "explode" but most of your body will remain intact - skin is capable of surviving a change of 1 atm. --Tango (talk) 12:47, 24 September 2008 (UTC)[reply]
Okay, let's summarise and separate out the hard science from the misinformation and speculation in the above responses:
  • High pressure: The human body is, chemically, a dilute solution of proteins and lipids in water. The bulk modulus of water is about 2x109 Pa. So to reduce the volume of a given mass of water by, say, 1% requires a pressure of 2x107 Pa or about 200 atmospheres. In other words, the body's cells and organs are, for practical purposes, incompressible. Saturation divers live and work in pressurised habitats at pressures of more than 5 atmospheres for days at a time. Ultra-deep divers at depths of more than 300 metres experience pressures of 30 atmospheres or more. The most dangerous effects of high pressure on the human body are associated with breathing gases under pressure, not with compression of body parts - see nitrogen narcosis and high pressure nervous syndrome.
  • Low pressure: Our article on uncontrolled decompression says "accidents in space exploration research and high-altitude aviation have shown that while vacuum exposure causes swelling, human skin is tough enough to withstand a drop of one atm. This assumes that the person doesn't attempt to hold their breath (which is likely to cause acute lung trauma), the limiting factor on consciousness then being hypoxia after a few seconds". So it is lack of oxygen that kills in vacuum, not the low pressure itself. Soyuz 11 depressurised slowly in orbit, shortly before re-entry, and the crew suffocated, but they did not explode. When Joseph Kittinger's glove seal failed during his Excelsior III balloon ascent, his hand was exposed to a pressure of 1/100 of an atmosphere for several minutes - this caused pain but no long-term damage. Our article on space exposure says that the maximum survivable time for exposure to vacuum is around 90 seconds.
  • Rapid decompression: a quick transition from high pressure to a lower pressure is the most dangerous scenario - see uncontrolled decompression. This not due to the expansion of body parts themselves but because of the expansion of gas in various places within the human body. Going from 1 atmosphere to very low pressure in less than half a second does not allow time for the lungs to empty of air, so causes lung damage. The effects of explosive decompression from 9 atmospheres to 1 atmosphere are graphically described in our account of the Byford Dolphin diving bell accident. Divers who surface too quickly can experience decompression sickness, due to the release of gases dissolved in body liquids and tissues.
Hope this help. Gandalf61 (talk) 13:08, 24 September 2008 (UTC)[reply]
Addendum to "Low Pressure": Its not just the lack of oxygen in the vacuum that kills you. The low pressure causes your blood to boil and so you lose what little oxygen may already be dissolved in your blood. The low pressure robs you of oxygen you have already inhaled! Also, I think the main article to point to should be space exposure. Otherwise, nice summary --Shaggorama (talk) 02:55, 25 September 2008 (UTC)[reply]

the first fortified peanut butter that is not plumpy nut

I am looking for any information on fortified peanut butter. Not the Plumpy Nut, but anything having to do with fortification, enhancement, any additions to etc. —Preceding unsigned comment added by G1963 (talkcontribs) 02:34, 24 September 2008 (UTC)[reply]

Density

I feel like this question is like the "which weighs more, 1 lb of bowling balls or 1lb of feathers", but I'm not sure.

"If you were to find the mass of 20 mL of seawater, would it be heavier, lighter, of the same as 20 mL of distilled water? And what does this mean in terms of density?"

I thought that the masses of each would be the same, and that the density of the seawater was more, but my friend told me I had it backwards. Is he right, or am I? —Preceding unsigned comment added by 69.16.88.147 (talk) 02:39, 24 September 2008 (UTC)[reply]

Without saying anything about seawater, we can reduce the two questions to one: if the a volume of X has more mass than the same volume of Y, then X is more dense. That's just the definition of density. To decide which substance is denser requires knowledge of the composition of seawater and what that means for density (or else looking it up). The important bit here is that adding a solute (e.g., salt) to a solvent (e.g., water) often increases the density because the molecules of the solvent clump up around those of the solute, reducing their volume. --Tardis (talk) 02:56, 24 September 2008 (UTC)[reply]
Agree with the first answer. Furthermore, think of buoyancy. When you lay in salt water, the buoyancy is different. You can float easier. Secondly, mythbusters did a show on quicksand. The result, is that you float extremely well, you have about 30% of your body above quicksand. Also, the ethereal air around us is very non-dense. When a skydiver jumps from a plain, that is like "sinking" because he is more dense than the air around him. Sentriclecub (talk) 03:00, 24 September 2008 (UTC)[reply]
Don't skydivers jump onto the plains, rather than from them? (sorry). :) -- JackofOz (talk) 05:38, 24 September 2008 (UTC)[reply]
Yeah you caught me, in the language of mathematics, there are no homophones. I'm a regular at the math ref-desk, only came here to post a question, but got too lazy. I'd be happy abolishing letters and using numbers only to communicate in writing. In which case, prime numbers could serve as vowels. I think I goofed because I think of the word plane as a mathematical term, oh well, I goofed, I should enjoy it. Sentriclecub (talk) 06:42, 24 September 2008 (UTC)[reply]
8 1, 8 1. -- 10 1 3 11 15 6 15 26.
9 & 7 5 20 & 9 20 Sentriclecub (talk) 10:03, 25 September 2008 (UTC)[reply]
I'd really like to see 1 lb of bowling balls! SteveBaker (talk) 23:50, 25 September 2008 (UTC)[reply]

Progressivity of electrical loads

If we attach a variable voltage source across a load, we will typically produce a fixed current for each voltage and obtain a current function . With a few exceptions, the function is strictly increasing, and if the load is unpowered clearly . For what broad classes of unpowered loads is the function convex, concave, or neither? (A pure resistance/reactance, of course, is the only convex and concave load.) Are there any reasonable components for which the current function is multivalued? DC and AC (considering the magnitude of current; see power factor) both interest me. --Tardis (talk) 03:22, 24 September 2008 (UTC)[reply]

A voltage source across a forward biased diode would produce negligible current until a certain voltage was applied (.7 volts for a silicon diode) then sharply increasing current for higher voltages, thus perhaps "concave" although the applicability of "concave" and "convex" is questionable. For a classic resistor it would be linear. Edison (talk) 05:48, 24 September 2008 (UTC)[reply]
Im afraid that what you say about diodes is entirely wrong, Edison. A semiconductor diode always has an exponential relationship of current to applied voltage (neglecting the small reverse leakage current). The current is given (approximately) by I= Io exp(qV/kT). This sort of poppycok bandied about by many people about diodes behaving as some sort of voltage dependent switch does nothing to aid understanding of electronics and in fact can lead to many poor designs in circuitry. The article Diode has a grossly misleading diagram of the I V characteristics of a semiconductor diode, implying as it does, that there is some magical transition between ON and OFF. There is NO such transition. (see the equation) It is worth saying also that the equation shows that the voltage across two similar diodes will only be equal if thier currents are equal. Hence the operation of so called bandgap references. Lecture over!--79.76.251.108 (talk) 17:31, 24 September 2008 (UTC)[reply]
Things like fluorescent lamps have negative resistance so are convex to a degree. Other things like normal incandescent lamps or real resistive loads have concave function as the higher temperature lead to higher resistance. --antilivedT | C | G 08:56, 24 September 2008 (UTC)[reply]
Anything with negative resistance is automatically "neither" since it can't have it at and can't be decreasing forever. The diode and the incandescent lamp are the two examples I had been able to come up with; I should have mentioned them. That the diode is only convex when forward biased reminds me also that positive and negative voltage need to be considered separately (anything unpolarized must have an odd current function and can't be nontrivially convex/concave everywhere). Thanks for the input — any other ideas? --Tardis (talk) 13:26, 24 September 2008 (UTC)[reply]
Have you seen our article on Negative resistance. It pretty much covers everything you need to know.--GreenSpigot (talk) 05:01, 25 September 2008 (UTC)[reply]
I have; I was just looking for more examples, to get a feel for the most common cases. --Tardis (talk) 23:36, 28 September 2008 (UTC)[reply]
Hmm. Have you seen Diac?--GreenSpigot (talk) 22:29, 1 October 2008 (UTC)[reply]
Also, what about thermistors?--GreenSpigot (talk) 22:31, 1 October 2008 (UTC)[reply]

Lag time and planetary alignment issues regarding laser transimission between Mars and the Moon/Earth

I'm doing research for a web comic, and I'd like to know, roughly:

1- The lag time for laser communications between the Moon/Earth and Mars.

2- The Sun is between Mars and the Moon/Earth for X amount of time. Time of year would be great but not necessary.

Also, I'm assuming that all lasers travel at the speed of light or thereabouts i.e. specific wavelength doesn't matter. Please don't trouble yourself looking for a formula(s) - I'm afraid physics equations are a bit beyond my reach.

Again, I'm looking for general estimates- please don't go through a lot of trouble calculating. Thank you so much!

Sevenzark 7 (talk) 05:36, 24 September 2008 (UTC)sevenzark_7[reply]

speed odf light 0.3 million km/s so divide the distances in the Mars and Moon article (normally given in million km) by 3 and you have the seconds. Easy to do it! But for the moon the estimation would be one second for one direction and there and back 2 seconds.--Stone (talk) 06:51, 24 September 2008 (UTC)[reply]
For Earth/Mars communication it varies depending on the time of year on both Earth and Mars. If we're both on the same side of the sun, I think it can get as low as three and a half minutes, or if they're on opposite sides of the sun it can get as high as twenty minutes. Roughly. APL (talk) 12:46, 24 September 2008 (UTC)[reply]
And starting around the mid-November, NASA will loose communication with the rovers/lander for 2 weeks, while Mars is directly behind the Sun. [5][6] -- MacAddct1984 (talk &#149; contribs) 13:34, 24 September 2008 (UTC)[reply]
I would imagine that laser communication from Earth to Mars would be really tough when they are more or less opposite each other with the Sun in the middle. Imagine pointing your super-sensitive light detector anywhere close to the sun and looking for a bajillionth of a candela of laser light energy. It's worth saying that the speed of light (which is the speed of the laser) and the speed of radio waves is exactly the same - so there is no particular advantage (or disadvantage) in using a laser versus a more conventional radio link. SteveBaker (talk) 00:11, 25 September 2008 (UTC)[reply]
(See bit rate :) Saintrain (talk) 01:09, 25 September 2008 (UTC)[reply]
The time of year during which the Sun will be directly between Earth and Mars is different every year, so it depends which year your comic story is supposedly happening. You could predict it, though, based on the future date and reasonnably simple calculations and MS excel. Some years the Sun will never be directly between the 2. --Lgriot (talk) 07:09, 25 September 2008 (UTC)[reply]

AWESOME! Thank you folks! Thanks SteveBaker I was wondering about radio waves. MacAddct1984 and Lgriot thanks for the date info! Sevenzark 7 (talk) 21:29, 28 September 2008 (UTC)sevenzark_7[reply]

Cell phone radiation and hard drives

Does cell phone electromagnetic radiation cause any harm (file corruption, data loss, etc.) to computer magnetic hard drives? If one were to put five cell phones (in use) on top of an external hard drive, would it do anything? —Lowellian (reply) 05:40, 24 September 2008 (UTC)[reply]

Short answer--no. Cell phones use low energy electromagnetic waves. If you have ever been in the airport and your computer was passed through a x-ray, that is about 10,000 times more energy per photon. Ionizing radiation is hopefully a blue-link. 99.9999999% of peoples computers survive the x-ray machine. However, the long answer is that my TV got slightly miscolored when I put my cell phone up to it. It needed a degauss button, but its an old tv, like 10 years old. But honestly, I'm supposed to be scientific, but I partly believe that cell phones shouldn't be held up to the ear when talking through them. Afterall, isn't the question about brain cancer the big elephant? I'm 24, and I own no cell phone! Hurray for me. Also, the neat part of your question... put five cell phones, is that electromagnetic radiation is not additive. In other words, if an apparatus is 60% of the energy required to emanate ionizing radiation, then a thousand of these operating concurrently will still not be able to emanate ionizing radiation. In the chemistry of photons... 50 joules + 50 joules + 50 joules + 50 joules does not equal 200 joules, but instead simply equals 50 joules. See photoelectric effect. Don't take my answer literally, I'm just regurgitating stuff from textbooks, i still find it hard to believe, and yet fascinating! Sentriclecub (talk) 06:34, 24 September 2008 (UTC)[reply]
I'm still somewhat confused though. I've tried holding up a cell phone in use to a stereo system and a television set, and it produces MAJOR audio and visual distortions, to the point that the audio/video is unlistenable/unwatchable. Considering that it can do so much to a stereo system and a television set, why would it by contrast do so little to a hard drive?
Lowellian (reply) 07:32, 24 September 2008 (UTC)[reply]
That's because the wires in your sound system acted as an antenna, receiving the signal of your cell phone, which is then amplified by your amps and made audible through your speakers. Not sure about video though since I have never experienced that before. Without the amplifier part (which amplified the few milliwatts of power from your cell phone to tens/hundreds of watts to drive your speakers) it simply does not have enough energy to disrupt your hard drive. --antilivedT | C | G 08:46, 24 September 2008 (UTC)[reply]
But if one's computer speakers are right next to one's computer (since they are, after all, computer speakers), by what you're saying, doesn't this then mean that these speakers will amplify the cell phone radiation right next to the hard drive? —Lowellian (reply) 04:12, 25 September 2008 (UTC)[reply]
Computer speakers have amps built into them (notice how they need mains to operate?) as line level signals from your sound card is simply not enough t drive the speakers. Try putting a cell phone next to a proper speaker with no amplification: you won't hear a single thing. --antilivedT | C | G 06:08, 25 September 2008 (UTC)[reply]
In addition, hard drives are completely encased in thick metal, which has Faraday effect to block a lot of EM radiation you can throw at em. Hard drives are designed to be extremely hard to damage - and very much succeed. They are made from solid aluminum (or something else) and are for all practical purposes dust-proof. They can protect the data they hold even when thrown to the ground or dropped from large heights (I think modern HDD can withstand 300g's of shock). With proper data recovery techniques data can be recovered from hard drives that have been underwater for days or in an intense fire. Running hard drives while open or with fingerprints even work (I've done it - although I wouldn't recommend it) Also, even pretty powerful magnets won't do jack to them. To properly degauss a hard drive you'll need a professional degausser (10 thousand dollars). Some lowly EM radiation isn't going to hurt it. --mboverload@ 06:54, 24 September 2008 (UTC)[reply]
I would question whether the thick metal of the average hard drive is effective as a Faraday cage, since it has gaps in it that enable an electromagnetic pulse to easily fry a hard drive.
Lowellian (reply) 07:28, 24 September 2008 (UTC)[reply]
Faraday cage with holes will still block out electromagnetic waves up to a certain frequency. If I'm not mistaken the highest cell phone frequency is 2100Mhz, which has a wave length of roughly 14 centimetres (5 and a half inches). I'm not sure about how small the gap has to be in relation to the wave length in order to block out that wave but unless you have holes 14 cm in size (bigger than your whole hard drive) it's probably blocked by the casing. It's the same principle the screens work in your microwave. --antilivedT | C | G 08:46, 24 September 2008 (UTC)[reply]
No. [7] - QED! SteveBaker (talk) 00:05, 25 September 2008 (UTC)[reply]
No, not quite QED. I actually thought about this particular argument before I asked the question; the reason that cell phones having internal hard drives does not prove that cell phones radiation doesn't affect hard drives is because presumably, even in the case that cell phone radiation did affect hard drives, a manufacturer might still put an internal hard drive inside a cell phone after taking care to specially shield this internal hard drive in a way that normal computer hard drives are not shielded. (I'm not saying that cell phones would harm hard drives; I'm just saying that this particular argument doesn't prove that cell phones wouldn't.) —Lowellian (reply) 04:10, 25 September 2008 (UTC)[reply]
Perhaps you should take a phone apart then :p. Compared to the millimetre thick hard drive enclosures the metal sheets inside cell phones for shielding are diminutive (not that thickness matter anyway). --antilivedT | C | G 06:08, 25 September 2008 (UTC)[reply]

four acceleration

the wikipedia article on 'four acceleration' states that the acceleration felt by an accelerating object is the derivative of proper velocity with respect to proper time. most everything else that I have read including other articles on wikipedia state that it is the derivative with respect to coordinate time. Em3ryguy (talk) 05:57, 24 September 2008 (UTC)[reply]

Proper time sounds right to me - then it's the acceleration that the object would actually feel. Can you give an example of an article that defines it wrt coordinate time? --Tango (talk) 12:37, 24 September 2008 (UTC)[reply]
thats what I thought too at first. but I did some figuring and the acceleration would indeed seem to be alpha=a*gamma^3. a is regular coordinate acceleration. the derivative of proper velocity (v*gamma) with respect to coordinate time equals alpha (according to the online calculator I used). the article on proper acceleration clearly and unambiguously defines it that way. http://en.wikipedia.org/wiki/Proper_acceleration#Viewed_from_a_flat_spacetime_slice Em3ryguy (talk) 20:07, 24 September 2008 (UTC)[reply]
I'm confused, where is the article that disagrees? --Tango (talk) 20:10, 24 September 2008 (UTC)[reply]
http://en.wikipedia.org/wiki/Four-accelerationEm3ryguy (talk) 20:21, 24 September 2008 (UTC)[reply]
Sorry, what article disagrees with that article? Apart from the strange fact that we have an article on 4-acc and on proper acc when they seem to be defined to be the same thing, I don't see any contradiction... --Tango (talk) 21:53, 24 September 2008 (UTC)[reply]
Actually, they're not quite defined to be the same - proper acceleration seems to be defined as a 3-vector, and 4-acc is that 3-vector with a null time component. Is that standard? I would have just called the 4-vector "proper acceleration" and avoided 3-vectors completely... --Tango (talk) 21:56, 24 September 2008 (UTC)[reply]
the article on four acceleration clearly states ' four-acceleration is a four-vector and is defined as the change in four-velocity over the particle's proper time'. the article on proper acceleration clearly defines proper acceleration as dw/dt. that is the derivative of proper velocity with respect to coordinate time.(four velocity and proper velocity being more or less the same thing) http://en.wikipedia.org/wiki/Proper_acceleration#Viewed_from_a_flat_spacetime_slice Em3ryguy (talk) 22:16, 24 September 2008 (UTC)[reply]
which contradicts the statement at the top of that article that says " The proper acceleration 3-vector, combined with a null time-component, yields the object's four-acceleration" which must be wrong. Em3ryguy (talk) 01:30, 25 September 2008 (UTC)[reply]
the article on four acceleration attempts to show that you can use it to calculate the instantaneous (and therefore infinitesimal) change in velocity. but since coordinate velocity, proper velocity, and rapidity are all the same at low speeds then I would think that any of them could do the same. but my math isnt good enough for me to be sure. thats why I'm asking you. Em3ryguy (talk) 20:19, 24 September 2008 (UTC)[reply]
if a stationary observer measures A's coordinate velocity to be v and B's velocity to be v+dv and both start at the origin at t=0 then at t=1 B's position is (v+dv,1). simply transform coordinates to A's frame to get (g(v+dv-v),g(1-v(v+dv))). the infinitesimal velocity from A's point of view is therefore g(dv)/g(1-v^2-vdv). which equals dv*gamma^2. divide the infinitesimal velocity by infinitesimal proper time [d(proper time)] to get acceleration from A's point of view. alpha=a*gamma^3.Em3ryguy (talk) 20:44, 24 September 2008 (UTC)[reply]
you might also want to know that dgamma/dt=v*a*gamma^3 (again according to the online calculator 1/sqrt[1-((v[t])^2)] http://calc101.com/webMathematica/derivatives.jsp#topdoit).Em3ryguy (talk) 20:58, 24 September 2008 (UTC)[reply]
you can also check that the derivative of proper velocity with respect to coordinate time is a*gamma^3. v[t]/sqrt[1-((v[t])^2)] http://calc101.com/webMathematica/derivatives.jsp#topdoit Em3ryguy (talk) 21:19, 24 September 2008 (UTC)[reply]
and of course. d(coordinate time)/d(proper time)=gamma. Em3ryguy (talk) 21:19, 24 September 2008 (UTC)[reply]
I don't follow what you're trying to show there, however it appears you are making a mistake with your coordinate transform. Relativistic velocities don't add simply like that, you need the velocity-addition formula. --Tango (talk) 21:53, 24 September 2008 (UTC)[reply]
I'm not adding velocities. its a coordinate transformation. if the coordinate in one frame is (d,t) then the coordinates (d',t') in a second frame moving at v is (g(d-vt),g(t-vd)). (assuming that their origins coincide at t=0) Em3ryguy (talk) 22:16, 24 September 2008 (UTC)[reply]
What is g? And that's not what you wrote, you had "v+dv-v" which is adding velocities (well, subtracting them actually, but that makes no difference). --Tango (talk) 23:55, 24 September 2008 (UTC)[reply]
g is gamma for coordinate velocity v. (d',t')=(g(d-vt),g(t-vd)) = (g(v+dv)-v(1),g(1-v(v+dv)) assuming c=1 Em3ryguy (talk) 00:06, 25 September 2008 (UTC)[reply]
gamma for velocity v is of course 1/sqrt(1-v^2) assuming c=1 Em3ryguy (talk) 00:52, 25 September 2008 (UTC)[reply]
I think I understand now. the 'a' in the article on four acceleration is the coordinate acceleration as measured by the nonaccelerating observer. only if that nonaccelerating observer is moving at the same instantaneous speed of the accelerating object does it equal the acceleration felt by the object (but, I think, thats true of coordinate velocity, proper velocity, and rapidity since they are all the same at low speeds). in all other cases it does not. the article probably isnt wrong but its certainly confusing. Em3ryguy (talk) 22:38, 24 September 2008 (UTC)[reply]
the article should read 'Therefore, within that co-moving inertial reference frame the four-acceleration is equal to the proper acceleration that a moving particle "feels"'. —Preceding unsigned comment added by Em3ryguy (talkcontribs) 17:25, 25 September 2008 (UTC)[reply]
proper acceleration on the other hand, as I understand it, should always be the same for all observers. just as proper time and proper distance are the same for all observers. Em3ryguy (talk) 22:40, 24 September 2008 (UTC)[reply]
the derivative of rapidity with respect to proper time is the proper acceleritaon. that is also clearly defined in the article on proper acceleration http://en.wikipedia.org/wiki/Proper_acceleration#Viewed_from_a_flat_spacetime_slice Em3ryguy (talk) 00:39, 25 September 2008 (UTC)[reply]
http://en.wikipedia.org/wiki/Lorentz_factor#RapidityEm3ryguy (talk) 23:35, 25 September 2008 (UTC)[reply]

I have edited the pages in question to what I believe is a less confusing form. I would appreciate it if someone would double check my work. Em3ryguy (talk) 01:43, 26 September 2008 (UTC)[reply]

Do candles melt when put in a microwave oven?

This question has been bothering me and my roommates during the whole summer while sitting on our balcony. Enjoying a good wine we were contemplating on the candles that were burning on our table: Do candles melt using microwave radiation? I voted for no, because candles are made out of wax and wax is not influenced by microwave radiation. A microwave oven utilizes the polarity of molecules, such was water, to generate heat inside the object that is inside the oven. As I have found out on Wikipedia, candles are made of paraffin wax, meaning a mixture of alkane hydrocarbons with a length between 20-40 C-Atoms. I am not a chemist, but I would guess that those long alkanes do not have a dipole moment? So, is my explanation correct? Ced22 (talk) 08:43, 24 September 2008 (UTC)[reply]

Having just put a small tea-light into a microwave oven on full strength for 30 seconds I can report that the candle did not melt nor did it feel any warmer than when it went in. Not very scientific but it's a start. Richard Avery (talk) 11:09, 24 September 2008 (UTC)[reply]
On the contrary, that is very scientific. Confronted with a hypothesis (candles heat up in a microwave) you made a prediction (your tea light would heat up in a microwave), tested it (put the candle in the microwave) and used the results to reject the hypothesis. My man, what you did was the definition of science! Plasticup T/C 23:45, 24 September 2008 (UTC)[reply]
Science is not about ad hoc experimentation. --98.217.8.46 (talk) 14:54, 25 September 2008 (UTC)[reply]
Science is all about planned experimentation. Do you mean to imply that Richard Avery's was ad hoc? --Kjoonlee 23:57, 26 September 2008 (UTC)[reply]
I think it's just what Karl Popper would have done. William Avery (talk) 20:28, 25 September 2008 (UTC)[reply]
Thanks very much for doing the experiment, Richard. We do not own a microwave oven, that's why we have not tried this test. Apparently the results support my theory. I did some further research, apparently alkanes exhibit very low polarity.Ced22 (talk) 11:36, 24 September 2008 (UTC)[reply]
Presuming this is an ordinary microwave that you don't wish to break, you might want to be careful with this sort of experiment. If your running your microwave oven with only something which does not absorb microwaves, you're basically running it without anything which as our article mentions, could cause you to burn out the magnetron. While this probably won't happen in 30 seconds, you should try it for too long. The way to avoid this would be to put a glass of water or something similar while doing the experiment Nil Einne (talk) 18:42, 24 September 2008 (UTC)[reply]
A microwave can only heat polar compounds, water being ideal. Candles are made mostly of long chain alkanes (see paraffin) that are certainly nonpolar, so they are unaffected by microwaves. Dielectric constant is essentially a measure of polarity. --Russoc4 (talk) 14:02, 24 September 2008 (UTC)[reply]
That would be Dielectric constant, or relative static permittivity. jeffjon (talk) 15:29, 24 September 2008 (UTC)[reply]

It's probably also worth noting that paraffin wax is, essentially, just a shorter version of the same sort of molecules that make up polyethylene, and polyethylene is widely used as a low-loss dielectric in many electrical applications. "Low loss", in this case, means that it absorbs very little energy from electrical fields.

Atlant (talk) 17:38, 28 September 2008 (UTC)[reply]

Thanks for all the clarifications and comments, guys! Ced22 (talk) 08:55, 29 September 2008 (UTC)[reply]

Floodwater in the bathroom

There was a big flood in Sueca (Com. Valenciana) last night due to rainfall, and some people said on the TV that they had floodwater coming up through the toilet etc. in the bathroom in flats higher up (not groundfloor). I live in a 3rd floor flat: could floodwater theoretically come that far up through the water system in the case of a flood, where exactly does it comes out and how could you stop it? Thanks for info, --AlexSuricata (talk) 11:00, 24 September 2008 (UTC)[reply]

If floodwaters are backing up through the sewers, then the way to shut that off would be to close a sewer valve if one exists or otherwise disconnect your outflow pipes. — Lomn 13:36, 24 September 2008 (UTC)[reply]
If you look out the third floor window and notice floodwater above the level of the third floor toilet, then it might be expected to rise out of the toilet. If the flood water is far below the level of the toilet, it's hard to explain why water would rise far above the level of the source (the floodwater) without work being done, and such a phenomenon would allow a perpetual motion machine consisting of a turbine to generate power from the fall of the water back to the level of the floodwater. If the drain were clogged, and the toilet were flushed, it would overflow, with or without a flood. If the building drain were somehow shutoff to prevent floodwater entering the building, and water were entering the drains from the plumbing pipes due to toilets flushing or water going down drains, or if somehow water collecting on the roof were overflowing into floor drains in the top floor, then water could overflow from third floor toilets. Edison (talk) 15:50, 24 September 2008 (UTC)[reply]
Floodwater can and does emerge from toilets that are higher than the level of the source. It happens when flowing water creates a surge of pressure in pipes. It has been known to happen from sea tides in the early days of Seattle, when toilets drained directly into the sea. It's the same principle as described in blowhole (geology). ~Amatulić (talk) 19:59, 24 September 2008 (UTC)[reply]
I do not believe that tides would cause such a "pressure wave" which would cause water to rise from a thrid floor toilet, presumably many feet above the water level. Consider that building codes (in places I am familiar with) toilets be connected to an upvent as well as a drain, so any pressure wave should go up that upvent to the atmosphere, and the water in the trap should stay in the trap. We have noted here previously, however, that extreme winds have caused pressure variations causing visible level changes in the toilet due to the pressure differential in the bathroom and at the rooftop terminus of the upvent. I just cannot envision the water rising up the drain pipe tens of feet above the level of the floodwater. The system of drains I would expect to dissipate the transient shock of rising water level and prevent the spike of energy required for such a slosh. Edison (talk) 22:22, 24 September 2008 (UTC)[reply]
Wouldn't this need to be considered as a momentum problem? Fluid is flowing downward in the sewer pipes from higher areas. That fluid has a definite momentum. At the same time, fluid is backing up from the overload in lower areas, thus flowing backwards with some lower momentum. When these flows meet, the momentum must be dissipated, presumably via a pressure surge upwards in whichever local pipes are handy.
And of course, if it's a rainwater flood and the rain is falling up on the hills and entering the sewer, if you're in the valley, expect the water to seek its own level, which will be above your toilet. Franamax (talk) 15:40, 25 September 2008 (UTC)[reply]

'foods that promote cancer and how they promote it'

i would like a detailed analysis of foods that promote cancer and how each of these foods help promote this disease41.219.253.218 (talk) 11:59, 24 September 2008 (UTC)[reply]

As in foods that are Carcinogenic#Carcinogens_in_prepared_food? 194.221.133.226 (talk) 12:47, 24 September 2008 (UTC)[reply]
You may want read the whole article. Carcinogens act in a variety of ways as mentioned there. And there are so many different carcinogens and potential carcinogens, many of which may occur in food that any complete detailed analysis is liable to be many, many pages long Nil Einne (talk) 18:24, 24 September 2008 (UTC)[reply]

Location of seminal vesicles

Is the seminal vesicles located between the rib cage and spine ? —Preceding unsigned comment added by 86.96.226.14 (talk) 12:14, 24 September 2008 (UTC)[reply]

Based on the diagrams in our cleverly named seminal vesicles article, I'd say no. --LarryMac | Talk 12:29, 24 September 2008 (UTC)[reply]


Rehanrazak (talk) 15:01, 24 September 2008 (UTC)[reply]

Whats happens when Global Warming comes to apex?

Does another ice age start on earth? If so, how will that happen? What do you think that most countries (ie. USA, UK, etc) would do? --Anilmanohar (talk) 13:05, 24 September 2008 (UTC)[reply]

This is largely beyond the scope of the Reference Desk, as we have no way of knowing what or when the peak of global warming will be. As such, no meaningful prediction of international response can be made. As for ice age trends, it's certainly the case that Earth has historically cycled between warmer and cooler periods. Whether human-affected warming is sufficient to break the cycle is again a question we cannot reliably answer. — Lomn 13:34, 24 September 2008 (UTC)[reply]
Venus might be a good model. Plasticup T/C 15:48, 24 September 2008 (UTC)[reply]
Note that climatically we currently are in an ice age, as there is permanent ice in the Arctic and Antarctic. We are in a slightly warmer interglacial, but the real hothouse mode of Earth is a lot warmer. On the plus side, that means that a complete runaway effect as on Venus is quite unlikely. But on the negative side, that means that we may push Earth into a very much warmer and very different state. --Stephan Schulz (talk) 18:13, 24 September 2008 (UTC)[reply]
Global warming doesn't exist. It's all related to sunspots and solar activity. See Maunder Minimum. 31306D696E6E69636B6D (talk) 13:30, 29 September 2008 (UTC)[reply]
Isn't it kind of weird that what you said is an opinion? This is science, for god's sakes! Mac Davis (talk) 23:44, 29 September 2008 (UTC)[reply]

the charge carried by 100 electrons

write in scientific notation the charge carried by 100 electrons —Preceding unsigned comment added by 76.189.131.73 (talk) 19:24, 24 September 2008 (UTC)[reply]

Done. Not sure it helps, since you can't see my notebook. Why don't you try it now? DMacks (talk) 19:32, 24 September 2008 (UTC)[reply]
1.2 * 10^-7 steves, this probably won't help either since you didn't specify the units and I just made these units up. (1 steve = 1.2 *10^-9 times the charge of an electron) You might want to try electron -- Mad031683 (talk) 19:48, 24 September 2008 (UTC)[reply]
I get about 8.3 × 1010 steves...are you sure your calculations are correct? :) --WikiSlasher (talk) 11:49, 1 October 2008 (UTC)[reply]

a wire carries 2.496E17 electrons

a wire carries 2.496E17 electrons (2.496 x 10^17) determine the charge [c.j] —Preceding unsigned comment added by 76.189.131.73 (talk) 20:04, 24 September 2008 (UTC)[reply]

Done. Algebraist 20:09, 24 September 2008 (UTC)[reply]
You may wish to read electron, specifically for finding the charge of an electron. —Cyclonenim (talk · contribs · email) 20:34, 24 September 2008 (UTC)[reply]

25A fuse

a fuse is rated at 25A.Will it fail if 3600C pass through it in 3 minutes —Preceding unsigned comment added by 76.189.131.73 (talk) 21:17, 24 September 2008 (UTC)[reply]

Check out the definition of current. --Tango (talk) 21:26, 24 September 2008 (UTC)[reply]
Q=It should be helpful--GreenSpigot (talk) 21:40, 24 September 2008 (UTC)[reply]
There is not really enough information to answer the question, if you want to be picky, since it is not specified whether the current flows at a constant rate. Far less charge than 3600 coulombs could blow the fuse if it passed through it in a very short while. If the current source were a large capacitor, most of the 3600 coulombs might flow in the first second. The question is also deficient in not specifying the time/current characteristics of the fuse, since a fuse is not a little magic machine that blows instantly when anything above its nominal rating passes through it. Some 25 amp fuses could carry significantly more than 25 amps for a short while, and others with different time curves would blow quickly at the same current overload. A fuse rated 25 amps should not blow if the current never exceeded 25 amps. If it is an intro science class question, you might just state that for the purposes of the problem solution, you assume the current flows at a constant rate, and that the fuse blows if more than 25 amps flows for a time approaching the 3 minutes. Then calculate the current (charge flowing per second) and compare it to the fuse rating. You will get a clear answer, given the assumptions. Edison (talk) 21:41, 24 September 2008 (UTC)[reply]
Is summer vacation over so soon? Plasticup T/C 23:38, 24 September 2008 (UTC)[reply]

Archie Frederick Collins

I am drafting have moved to main space a biography of this prolific science writer and early radio experimenter (claims to have made voice transmissions in 1899, was a business partner of Nathan Stubblefield) at User:Edison/Archie Frederick Collins Archie Frederick Collins. He used "A. Frederick Collins" as his name in publications. He wrote about 100 books on science, technology and hobbies, numerous encyclopedia articles and over 500 articles in technical magazines and newspapers from about 1900 to 1946. His books often bring handsome prices on Ebay and some are still in print. He wrote the first edition of "The Radio Amateur's Handbook" in 1922, still in print in its 82nd revised edition. He was born in Indiana January 8, 1869, but neither Who's Who (which carried a bio listing and a listing in "Who Was Who") nor the Library of Congress, in its bibliographic index, has a date of death. He seems to have died sometime after 1946, since that was the date of his last book. He was a resident of New York City and of Congers, New York. If anyone has access to New York state vital statistics to find a death certificate, or to any Rockland County, New York or New York City local history sources or probate records , or for an obituary which does not show up online, or any Florida sources (where he once had a residence) I would be grateful. A "real life reference librarian" could not turn up a death date in any bio compendium. His wife was Evelyn (or "Eva Lena") Bandy Collins and they had one son, also an author, Virgil Dewey Collins (born 1898). Thanks. Edison (talk) 21:28, 24 September 2008 (UTC)[reply]

Well, I have one teeny-tiny scrap for you:
http://www.daggy.name/cop/bkofdead/obits-co.htm narrows his death date to between 1949 and 1954 and says that he dies in Congers, NY.
The 1954 date is somewhat backed up by http://www.ibiblio.org/ccer/1926a1.htm which points out that the copyright of his book "A Bird's Eye View Of Invention" was renewed by Thomas Y Crowell Co on 20th Jan 1954. The annotation after the renewer's name is "(PWH)" which stands for "Proprietor of a Work made for Hire". There is another annotation for "Proprietor of a Posthumous Work" - so perhaps that is evidence that the author was still alive when his publisher renewed the copyright...but that's pretty thin evidence.
SteveBaker (talk) 23:38, 24 September 2008 (UTC)[reply]
The "Daggy" site seems to be guessing. The copyright renewal site might prove something, but it is hard to interpret. I do not see a "PPW" for Collins there, which would at least give a year by which he had died. I believe his publisher was absorbed by other companies, so their records may be unhelpful. I have looked for this information for several decades. The lack of a New York Times obituary for a frequent contributor to that paper suggests that the family wanted to keep it low-key. He was sort of the Mr. Wizard of the pre-television age, with many books on science for children and how to build neat things. Someone of his prominence could easily have had obituaries in papers such as the NY Times, but no joy there. A death certificate from the NY vital statistics office would be ideal. I really need a reliable source for a date of death so I can publish the article in mainspace. Edison (talk) 04:54, 25 September 2008 (UTC)[reply]
In the real of copyright renewals, it looks like here [8] and here [9] that A.F. Collins renewed (as author) the copyright of "The book of the microscope" on 7 August 1951, at the age of 82. Agreed? So his demise must have been after that date, if the record is accurate. Edison (talk) 05:18, 25 September 2008 (UTC)[reply]
Then 16 December 1955 there is the copyright renewal of a 1928 title "Boys' and Girls' Book of Indoor Games" renewed by Carolyn Collins Vilk, "next of kin," [10] , with the implication he had passed on or was incapacitated by that date. 23 Feb 1955 there is a copyright renewal [11] of "The book of puzzles", a 1927 book, by Myra K. Collins, listed as "executor of the author," so he seems to have died by then. Collins seems to have died circa 1951-1955. That would at least make a vital records search cheaper, since they charge by the time span. .Edison (talk) 05:38, 25 September 2008 (UTC)[reply]
Oh! Nice detective work! I didn't think to look for copyright renewals of his other books. But I strongly agree - "next of kin" certainly implies either death or at least a terminal condition. This is strong evidence for a range from 1951 to 1955. It's really bizarre that hundreds and hundreds of Google hits say "1869-" with no death date. Collins sounds like a really interesting guy though - I look forward to reading an article on him. Invented the mobile phone and then went to jail for fraudulantly advertising it - came out of jail and couldn't get a job so he writes dozens and dozens of truly excellent books...an interesting guy evidently. SteveBaker (talk) 23:47, 25 September 2008 (UTC)[reply]
I found several books on communications, and on popularized science, which had some discussion of the significance of Collins' research and writing. I have moved the article from my sandbox to main space. I still feel that the combined effort of Wikignomes should be able to find a more precise date of death for such a well known author than "circa 1951-1955." Edison (talk) 22:37, 26 September 2008 (UTC)[reply]

It would be great if a few experts in the fields of electronics, physics, Tesla's theories and inventions would look this article over. It certainly seems a most exciting issue, as the size of the article, its content, the strong opinions and some mentioned sources on the talkpage suggest that Tesla's concept of wireless energy transmission of industrial-level electric energy was and remains very feasible, but especially as they all suggest that Tesla obviously knew about stuff such as waves in plasmas, magnetohydrodynamics, the ionosphere, ELF transmission communication, intentional telluric current, Schumann resonances, planet earth's self-capacity and its use as a cavity resonator, and Zenneck waves as far back as around and shortly before 1900, and that his only problem was financial support as soon as his entrepeneur investors found out that he intended to provide free electricity out of thin air for everyone on earth as a quasi-socialist public service; J. P. Morgan asked Tesla, "Where do I put the meter?", but Tesla neither knew nor cared. --80.187.125.4 (talk) 21:31, 24 September 2008 (UTC)[reply]

Despite Tesla's major contributions to electrical knowledge such as advances in alternating current motors, robotics, radio, high frequency/high voltage alternating current, and high speed turbines, he had some scientific ideas that have not gained acceptance, or which have been disproved, and is often given credit for understanding issues which he may not in fact have followed, and for having reduced to practice things like "death rays" which were only wild talk in his old age for the benefit of credulous reporters. Edison (talk) 21:47, 24 September 2008 (UTC)[reply]
Edison, you're just jealous. Face it, you lost the "current war" - it's an AC world now. ;) Franamax (talk) 12:15, 25 September 2008 (UTC)[reply]
I guess I can refute you on at least one call: Tesla's diary entries from as early as 1899 document his understanding of the ionosphere, see Tesla, Nikola, "The True Wireless". Electrical Experimenter, May 1919. It was detailed enough for him to predict the correct ELF frequencies required, half a century before their practical military use, as based upon what's now called Schumann resonances. Note that the radiation Tesla was about to utilize at Wardenclyffe was not Hertzian in nature but in fact Zenneckian, as he was openly saying that he was dismissing Hertzian radiation for far-distance wireless energy transmission and going for Zenneck waves instead, and thus the waves would be propagating basically in plasma, in Tesla's case particularly using air, water, and the planet earth itself as natural media conductors (basically like solar wind plasma). Also remember he'd found a way to draw electric energy from thin air years ago in Colorado Springs already, which was actually the cosmic background radiation, as well as demonstrating wireless eletric energy transmission through the air at a level high enough to fully power head-sized lightbulbs that could be carried by hand hundreds of feet around without glowing any dimmer as early as 1891, as that's the year when he demonstrated it at the American Institute of Electrical Engineers as well as the National Electric Light Association and the Franklin Institute.
And calling his Teleforce design for a particle beam weapon a "death ray" makes it sound more ridicilous than both thes article particle beam weapon as well as teleforce obviously make his device out to be. The basic device you need for particle acceleration is a van de Graaff generator which had been around for half a decade when Tesla started pushing his teleforce design, and contemporary news coverage read that he was either to use a van de Graaff generator or an updated version of his magnifying transmitter. The teleforce obviously was close enough to production that he tried to sell it to several Western governments before WWII.
The other buzzwords mentioned above certainly weren't around by 1900 so it's no surprise that Tesla didn't use exactly these terms back then when explaining what his plant at Wardenclyffe was to be doing. Also, the Wardenclyffe plant certainly isn't much to do with his "later years", as he was still four decades from his death. --80.187.125.4 (talk) 02:14, 25 September 2008 (UTC)[reply]

How to store things on an earth floor yet keep them dry

I want to store boxes of books etc. in a small outbuilding which has an earth floor. I have done this before in the past, and I found that neither old carpets or sheets of plastic stopped the moisture from the earth seeping through and making the books damp and subject to insect attack. Can anyone suggest a cheap easy readily available thing to put between the earth floor and the books to keep them dry please? Or some other way of seperating them from the floor that supports them? Aluminium foil for example would be too fragile. Thanks. 78.147.10.10 (talk) 21:58, 24 September 2008 (UTC)[reply]

A wooden pallet or plywood supported by thich planks can keep the books up off the ground. If they are on the ground, even on plastic, the coolness of the ground could cause condensation. They still need protection from dust, leaky roof, rodent and bug infestation. A metal clothes cabinet up off the ground might help keep mice and bugs out. There are humidity absorbers available at hardware stores which have a chemical such as calcium chloride in them to absorb moisture. Some can be heated periodically by taking them and plugging them into an electric outlet elsewhere to get rid of the absorbed moisture. Sheets of old carpet or carpet underlayment (as thermal insulation) on top of plastic (as a moisture barrier) could also insulate from the coolness of the ground.Edison (talk) 22:12, 24 September 2008 (UTC)[reply]
You could store your books in one of those vacuum-sealed bags (I'm thinking of Space Bags...can you believe we have an article about them?!). These are huge polythene bags - you could store dozens of books in them - with a fitting that lets you hook up a vacuum cleaner and suck out most of the air. Alternatively, you could just stick each book into an individualised 'ziplock' baggie. The result should be pretty much resistant to everything...although if they're going to be there for a long time - you'd want to keep them out of sunlight. SteveBaker (talk) 23:07, 24 September 2008 (UTC)[reply]
A pallet would be great. You can sometimes get old or broken pallets for free from large retail stores or transport companies. As for the bugs use insecticide under the pallet. Mieciu K (talk) 23:39, 24 September 2008 (UTC)[reply]
I think the important thing is to get them up off the ground, wooden pallets as Edison suggests would be quite good. Once they're off the ground, you can just put them in bin bags. --Tango (talk) 23:47, 24 September 2008 (UTC)[reply]
Some plastic container, even a large plastic bin, would be good. Mice and bugs should be kept out, once the moisture problem is solved. Edison (talk) 04:49, 25 September 2008 (UTC)[reply]
Raising the books off the floor is key, so that there's air circulation underneath. When I had to store my books for several months, I put them in boxes (from the liquor store, which gives them away free) and bought 4-ounce bags of silica gel. They were rated as giving 3-month protection inside of the sealed unlined cardboard boxes. I would recommend the same approach here, except using hard plastic containers would be better to keep away the critters. Do throw in some desiccant though. Once your books are wrecked, they're wrecked forever. Franamax (talk) 12:05, 25 September 2008 (UTC)[reply]

Thanks, I do not think using pallets would be a good idea as the wood would get moist and thus be a magnet for various insects and rots (where I live the soil is moist - perhaps in desert areas in the US it is not). Plastic containers may work but I would need very many of them, which would be expensive. I was hoping someone might suggest some sheeting which is truly waterproof, as polythene is not. Some non-ferrous metal sheeting may be best, if I can get some cheap. 89.243.119.61 (talk) 19:56, 28 September 2008 (UTC)[reply]

You are almost dead-wrong here. The key is to allow air circulation between the earth and the books. We don't all live in the desert (or the US) - a skid (pallet) will work fine on dirt, that's what they do, but if you're worried, put the skid on four bricks so it doesn't touch the floor. Metal sheet will provide an equally good condensation surface, you need the air gap. However, if you are proposing to store books unprotected in an enclosed space with a damp-earth floor, you'd best give them away now. You will need all the factors - raised, in plastic boxes, with dessicant. Franamax (talk) 11:44, 30 September 2008 (UTC)[reply]

Both the wood of the pallet and its supporting bricks (unless engineering bricks were used) would gradually wick up moisture from the soil. It may be ok if supported off the soil by tin cans, for example. I do not see why a metal sheet would have more condensation that any other surface. Pond-liner may be a truly waterproof sheeting, but I think its quite expensive and not readily available. You are right that ventilation is needed. 78.149.137.49 (talk) 01:24, 1 October 2008 (UTC)[reply]

You're on the right track. Minimizing the contact surfaces is important, and airflow is important. The vertical edge of a pallet sitting on a brick will indeed wick some moisture but you are now dealing with only a few square/linear inches as opposed to putting down some carpet right on the dirt. Look at the moisture path - up a brick, to the bottom of the pallet vertical, up the vertical, across the surface of the pallet, into the books/boxes - that's a long path. My point about a metal sheet (or a pond-liner) is that any continuous surface will provide an avenue for moisture creep, the moisture will come out of the damp air and move along the surface.
Those waxy cardboard boxes would also provide a good sealed environment for the books (still should be kept off the ground though), and I'd still recommend a desiccant inside. Sorry for my passion on this subject, I just hate the thought of perfectly good books getting ruined! :) Franamax (talk) 01:41, 1 October 2008 (UTC)[reply]

DOes anybody know what's the colour of Pluto's sky. Could it be something else besides black like purple-black or dark blue. I thought it has thin atmospher.--57Freeways 22:37, 24 September 2008 (UTC)[reply]

According to Pluto, its atmosphere is more than a hundred thousand times thinner than Earth's atmosphere. I would expect the sky to be totally black to the human eye. Algebraist 22:41, 24 September 2008 (UTC)[reply]
And even if the atmosphere were denser - the sun is so amazingly dim at that distance that very little light would scatter from it - so yeah - VERY black! On the other hand, the lack of light pollution and the clarity of the atmosphere should make for astoundingly beautiful stars - and that HUGE moon...WOW! SteveBaker (talk) 23:00, 24 September 2008 (UTC)[reply]
Given the distance, I'd expect Charon to be pretty dark too, if not completely black.--Fangz (talk) 23:26, 24 September 2008 (UTC)[reply]
This artist doesn't think so! Wow. Plasticup T/C 23:34, 24 September 2008 (UTC)[reply]
Charon's albedo is about 3 times that of our moon and it's more than 10 times closer to Pluto than our moon is to us, so that will compensate a bit. --Tango (talk) 23:50, 24 September 2008 (UTC)[reply]
Being nice and shiny can only do so much. Pluto gets 0.06% as much sunlight as the Earth. Dragons flight (talk) 02:48, 25 September 2008 (UTC)[reply]
Hmmm - let's toss some numbers around: so Pluto orbits between 30 and 50 AU's from the Sun (the Earth orbits at 1AU). The sunlight will get dimmer as the square of the distance - so the sun appears to be between 900 and 2500 times dimmer on Pluto and Charon than it is here. If Charon is 3 times shinier - then it would look 300 to 800 times dimmer than our moon - although the lack of an atmosphere on Pluto would give that a bit of a boost. Being closer to Pluto won't really make it look brighter because it's not remotely like a "point source" like the sun is at that distance - it's 3 times smaller in diameter than our moon - but it's 10 times closer - so it's going to look about three times bigger than the moon looks from the Earth - that would be quite something to see...but yeah - it would be pretty dim even when it's a "full-moon" (er "full-Charon"?). Pluto and Charon are tidally locked - so if you could see Charon at all from your location on Pluto, it would stay put in the same place in the sky all day and all night. Then, just for fun, you'd have two other teeny-tiny moons (Nix and Hydra) to look at...they are really pretty tiny rocks though - as dim as they are, you might have a hard time spotting them with the naked eye from the surface of Pluto. SteveBaker (talk) 02:51, 25 September 2008 (UTC)[reply]
Good point about it being closer not making a difference - I was thinking about the total amount of light you would receive from the moon, rather than the brightness (which is defined as being per unit area). --Tango (talk) 15:07, 25 September 2008 (UTC)[reply]
This thread inspired me to do some reading about the planets and moons of the solar system last night. The view from Phobos sounds absolutely stunning. --Kurt Shaped Box (talk) 17:00, 25 September 2008 (UTC)[reply]
That is what I love about the Ref Desk. In answering questions and in reading replies, I am taken to parts of the encyclopedia that I might otherwise never visit. Plasticup T/C 21:53, 25 September 2008 (UTC)[reply]
Yes, indeed. Take a bunch of inquisitive people and the whole of human knowledge in an easy-to-read form and you get something kinda beautiful. I love the part that Phobos will soon (well, within 11 million years) break up due to tidal forces and Mars with have a spectacular (if short lived) low-altitude ring system...followed soon after by 1013 tonnes of rock raining down from the sky onto a line precisely around Mars' equator....WOW!....SciFi authors need to know this stuff - stories need to be written! SteveBaker (talk) 23:35, 25 September 2008 (UTC)[reply]
Of course, they'll need to change the 11 million years to 11 days, Phobos to our Moon and Mars to Earth, and then remove any resemblance to valid physics. They then need to blow up the Moon with a few nukes (ignoring the fact that the moon breaking up is the problem in the first place giving them extra credit on the valid physics thing) throw in an unrelated romance and it will be a box office hit, for sure! --Tango (talk) 23:54, 25 September 2008 (UTC)[reply]
Nice. I'd read an sf book in which Phobos gets crashed into Mars, but I didn't know it could happen naturally. Algebraist 23:58, 25 September 2008 (UTC)[reply]
Replying to Fangz, even on a moonless night without cloud cover, the earth is not pitch black (very dark true, but not totally black). Starlight and Zodiacal light surely would count for something, but I agree the artistic picture takes liberties with appearances (perhaps a long camera exposure rather than naked eye). See also Olbers' paradox for why not very bright from starfields. David Ruben Talk 19:48, 1 October 2008 (UTC)[reply]

How often does this dysfunction happen? How large can a Macropenis be? What are the possible implications other then the inability to have a normal sexual intercourse? Mieciu K (talk) 23:35, 24 September 2008 (UTC)[reply]

There is very little information about this in the literature. I think that use of this term is confined to boys who are undergoing puberty or who have not yet reached puberty. See human penis size. Axl ¤ [Talk] 12:09, 25 September 2008 (UTC)[reply]


September 25

Planets, homework for MY 5th grader, can't find in packet, please help!

This planet has rings and 15 moons.

This planet has 18 moons and 7 rings.

This planet has 8 moons, 2 that can be seen by telescope.

This planet is 93 million miles from the sun. (by ang)

this planet's orbit crosses Pluto every 248 years. —Preceding unsigned comment added by 75.132.46.189 (talk) 00:03, 25 September 2008 (UTC)[reply]

Uranus, Saturn, Neptune. Earth is 93 millions from sun, what you think? now Jupiter have at least 63 moon,s at least 60 for Saturn, at least 27 for Uranus, at least 13 for Neptune. Your book must be at least 10 years old.--57Freeways 00:14, 25 September 2008 (UTC)[reply]


So according to my daughter's book/packet: ( she's with grandma and i'm working) This planet has rings and 15 moons. JUPITOR

This planet has 18 moons and 7 rings. SATURN

This planet has 8 moons, 2 that can be seen by telescope. NEPTUNE

This planet is 93 million miles from the sun. (by ang) EARTH

this planet's orbit crosses Pluto every 248 years. EARTH

Yes, that was right 10 years ago if you look for a book copyright in 1996. But now scientist keeps finding more moons, and numbers of moons keep multiply. Jupiter, Saturn, Uranus, and neptune is a gas giant when you try to land you can't because there is no place to land. See the article about Jupiter.--57Freeways 00:51, 25 September 2008 (UTC)[reply]

Use this table to help.--57Freeways 00:57, 25 September 2008 (UTC)[reply]

Earth is definietely 93 million miles away from the sun.--57Freeways 00:58, 25 September 2008 (UTC) this planet's orbit crosses Pluto every 248 years. I'm pretty sure this is Neptune.Avnas Ishtaroth drop me a line 01:20, 25 September 2008 (UTC)[reply]

This is the Science desk - so it important to be 100% clear and scientific about this:
  • This planet has rings and 15 moons.
  • This planet has 18 moons and 7 rings.
  • This planet has 8 moons, 2 that can be seen by telescope.
There are no planets in the Solar System that fit any of those descriptions. I'm pretty sure none of them did even 10 years ago when this was printed. All three gas giants have uncountable numbers of rings and we'll still be finding more teeny-tiny moons 20 years from now! The number of moons that "can be seen by telescope" is a crazy statement. Where is the telescope? How the heck does the questioner think we found the other six moons if not by staring at them through a telescope?! Stupid, stupid, stupid. All three questions are beyond redemption. Even ten years ago, it was very apparent that we'd be finding more moons all over the place and that the question would likely be incorrect before the ink dried on the page. Worse still - it misses the beauty that is these ring systems. Jupiter has a donut as well as its rings. The hundreds of separate rings around Saturn are sometimes braided together and twist and un-twist as little 'nursemaid' moonlets orbit through the debris. It misses the point that moons and moonlets extend out from these large gas giants in uncountable numbers.
  • This planet is 93 million miles from the sun. (by ang)
I don't know what "by ang" means - but the only planet that is at approximately that distance is the Earth (which varies from 91.4 to 94.5 million miles from the sun - with an AVERAGE of 93 million miles). The "(by ang)" thing...WTF??
  • This planet's orbit crosses Pluto every 248 years.
Well - this is clearly INTENDED to elicit the answer "Neptune" - but it's not remotely correct. Pluto NEVER crosses Neptunes orbit - nor vice-versa! Not even close! It only looks that way on a typical top-down view of the solar system such as a teacher writing a textbook who knew less about the solar system than most of his students would see when struggling to write a quiz! If you look at the system in three dimensions (see picture to the right here) - you see that Pluto's orbit is so tilted that it never comes anywhere near Neptune. In fact, the closest that Pluto and Neptune EVER get to each other is about eleven times the distance from the Earth to the Sun! Pluto actually gets closer to Uranus than it ever gets to Neptune. But even if we're super-generous and we agree to use the classical "top down" textbook view - Neptune has an orbital period of 164 years and it "crosses" Pluto's orbit twice in each loop around the sun...so it crosses it TWICE in every 164 years - not ONCE in every 248. What they are thinking of is that Pluto's orbit is 248 years - but it too "crosses" the orbit of Neptune twice in each orbit - so even if the question asked which body crossed the orbit of Neptune every 248 years - they'd still be wrong. This question is SO far off being right...it's absolutely unbelievable. The orbital mechanics of Pluto and Neptune have been known for at least 60 years - so we can't even accept that the answer is a bit out of date...it's flat out WRONG.
So, this very, very sad, sorry excuse for a quiz is some how managing to teaching kids at least six or seven incorrect facts in the space of just FIVE questions - and the only one that's remotely right is confusing! Heck, I'm beginning to think one of those Creationist loonies could have done a better job of this! ("Which planet did God name after Donald Duck's dog?")
When people wonder why standards for science education is declining...this is the reason. Please tell your child's head-teacher that he/she should be ashamed to be pushing lies and half-truths onto the kids. It would be better not to teach them anything on that quiz than to teach them lies and half-truths.
SteveBaker (talk) 03:33, 25 September 2008 (UTC)[reply]
Steve, how disrespectful to those of faith. Everyone knows that Pluto is not a real planet. On top of which, Pluto was Mickey Mouse's pet!! Have you even read the bible? :) Franamax (talk) 14:44, 25 September 2008 (UTC) [reply]
Yes, I know - I was trying to cleverly cram as much disinformation into a single sentence as possible...kinda like that last question in the quiz. Not only is Pluto not a planet anymore and Pluto is indeed M.Mouse's pet, not D.Duck's. But also: The name came from an 11 year old girl called Venetia Burney, not "God". The "dwarf planet" is named for the Greek version of Hades. It was a full 10 years later that the cartoon dog was named after the planet, not the other way around. Therefore, the only appropriate answer to my question is, of course "Goofy"! (Also - I'm one of a decreasing number of people who actually sat down and read the Bible from beginning to end...it's nearly as funny as Goofy.)  :-P SteveBaker (talk) 23:20, 25 September 2008 (UTC) [reply]
Yes, all as told in 2 Disney 4:3-19, you have read it! :) I've read the whole thing too, twice actually. I particularly liked the sequel, "God: The NEW Testament" - it had a lot less sex (no "begat"s) but some really awesome special effects (Revelations). Franamax (talk) 02:54, 26 September 2008 (UTC)[reply]
All of that namby-pamby helping out the poor and needy thoughout the sequel is all very well - but I missed the whole killing off the youngest kids of an entire civilisation, raining blood (that stuff stains!), annihilating the entire ecosystem of the planet, screwing up people's brains so they all speak different languages...all great "vengeful god" stuff! SteveBaker (talk) 04:16, 26 September 2008 (UTC)[reply]
To be fair, Steve, there's at least a reasonable answer to the telescope bit. Many smaller moons were discovered via the Pioneer and Voyager missions, and were (at the time) quite invisible to Earth-bound telescopes. Given the HST and recent developments in adjusting for atmospheric interference, I doubt it holds any longer -- certainly the discovery of minor satellites around Pluto and such cast it into great doubt. But there's absolutely a way to discover a tiny moon without a telescope. — Lomn 15:54, 25 September 2008 (UTC)[reply]
Well, a lot depends on whether you consider the cameras on Pioneer and Voyager to be "cameras" or "telescopes". Wiktionary says "A monocular optical instrument possessing magnification for observing distant objects, especially in astronomy." and Telescope says "A telescope is an instrument designed for the observation of remote objects and the collection of electromagnetic radiation.". The 1500mm lens that Voyager used for it's narrow field camera certainly falls under both definitions. And with that much magnifications, it's definitely what I'd call "a telescope". HST had been in use for 6 years before those questions were printed and it can image even the two tiniest known moons of Pluto - those are only 60 km's or so across and 30 to 50 AU's away. If the supposed answer is "Neptune" then most of it's smaller moons are over 150km across so HST can certainly image them. SteveBaker (talk) 23:20, 25 September 2008 (UTC)[reply]
I read this as an ambiguous distinction also. Can be seen by telescope == depends on the telescope. Ground-based, space-based, set up in your back yard? Presumably the quiz is meant to be answered based on reading of a specific textbook - so the answers don't have to correspond to physical reality, they just have to satisfy the teacher that you've read, integrated and understood the supplied materials. That's really the basis of early education.
Followup question though: once the various moons and rings were discovered by exploration craft, haven't they been subsequently observed and tracked by ground-based instruments? Franamax (talk) 02:54, 26 September 2008 (UTC)[reply]
(Read, integrated and understood a lot of irrelevent and incorrect garbage from an outdated textbook...yes)
Probably, yes. The tricky part of spotting moons is that they are very small and dim and they move quite quickly. You need LONG exposure times to capture a picture of a very dim object. Once you know the moon's orbital parameters, you can program your ground-based telescope to track it for hours accumulating tiny amounts of light and after enough time you'll have a decent picture. But the snag is that you can't FIND a moon that way - if you don't know where it is and how it's moving, you just can't image it. So I think that's probably why the spacecraft-based discoveries were initially important. Another part of it is that accumulating light in ground based telescope like this is great for doing things like spectroscopic and redshift analysis - but it's not so good for seeing craters and interesting surface features - for that, you need a spacecraft nearby. SteveBaker (talk) 04:01, 26 September 2008 (UTC)[reply]
To a certain extent, the precise accuracy of information taught to a 5th-grader is not relevant, although it would be nice. Most people will get through life just fine with only a hazy understanding of the solar system - as long as they know planets orbit the sun, moons orbit planets, the whole thing is really big - knowing Pluto's orbital eccentricity doesn't help butter your toast. Learning how to learn is always the aim, and more precise facts can always wait 'til later schooling. And of course, now there's Wikipedia, where you can check to find out that the current number of known moons of Neptune is "my firend Janey is so awesome".
You need to also keep in mind the incredible cost of new textbooks and learning materials plied by various consultancies; also the fact that many elementary-school teachers by and large didn't learn any science themselves beyond fifth-grade. Franamax (talk) 06:37, 26 September 2008 (UTC)[reply]
I have to disagree with most of that. Things learned at an early age tend to stick very much. Why do you think oodles of unqualified (and indeed many qualified) people were interested in the status of Pluto? We don't need to teach all the details, of course, but we should make sure that we do not teach things that are plainly wrong. Yes, "learning to learn" is one aim. But such learning is much easier if you manage to get pupils interested in the material - and not disillusioned when they fact-check it somewhere else. And I dispute the "incredible cost of new textbooks and learning materials". For the price of one BGM-109 Tomahawk, we could buy about 10000-20000 elementary school textbooks. And unless my maths is wrong, you could bury each inhabitant (not every pupil!) of the US under 1000 textbooks per year calculating just the direct cost of the Iraq War. Of course, as a society we need to decide where our priorities are, but I would put more into the education of people and less into killing them. --Stephan Schulz (talk) 09:36, 26 September 2008 (UTC)[reply]
You are assuming the textbook is old. I've had the opportunity to review drafts of a middle school science text currently in preparation and it contained some ridiculous inaccuracies (on par with those above) even though it was being written now. The problem, as I see it, is that much of the work in preparing science texts for elementary and middle school education is being done by people whose primary background is either education or writing. Such people seem to assume that if they graduated high school they obviously know enough science to teach it to young people, and that blind faith leads to some ridiculous errors. Thankfully there are some scientifically trained reviewers, but all in all the state of science texts below at least the 9th grade level can be pretty abysmal. It's not uncommon to see old untruths regurgitated because that's what the writer learned when they were in school and they don't have the science background to know when knowledge has evolved. Dragons flight (talk) 09:53, 26 September 2008 (UTC)[reply]
It's instructive to read what Richard Feynman (my personal hero) thought about science textbooks - he took on the job of reviewing science books and was eventually defeated by the sheer frustrating task of fighting the system and trying to fix problems. I've tried (and failed) to get gross errors removed from several math and science textbooks in Texas. Even when the books were revised and reprinted - they ignored my error reports. In one case, a book has had the same error through three reprints despite three long and carefully reasoned/documented letters of complaint to the author, the publisher and the school authorities. I've had better success in getting my kids' grades fixed when he answered a question 100% correctly and was marked down because the official answer was hopelessly wrong. Some of the problems were pretty serious - for example, in geometry, it was exceedingly difficult to get the school authorities to agree that a square is a rectangle and a parallelogram and a rhombus...or that an equilateral triangle is an isosceles triangle and an acute triangle. Those are not small matters - the idea that because something is a member of a specific class, that this somehow removes it from a more general class is a severe academic error that can affect someone's thinking for years to come. Those kinds of problems go well beyond accidentally teaching that Uranus has the wrong number of moons. However, there is no excuse for having ANY errors in a textbook. SteveBaker (talk) 12:58, 26 September 2008 (UTC)[reply]

Btw, can someone tell me what a "packet" is, as used in this question title? Packet makes me think of cereal boxes, not textbooks (unless that is where the incorrect information comes from?) Gwinva (talk) 04:38, 26 September 2008 (UTC)[reply]

Sometimes, especially in grammar schools, teachers hand out material (photocopies of articles, worksheets, selected pages in books, etc.) in bundles or "packets" (often in manilla envelopes) and teach from that material rather than from textbooks. That's what's being alluded to here. Of course, it ought to be easier to keep such material up to date! - Nunh-huh 05:17, 26 September 2008 (UTC)[reply]
Thank you! I suspected that might be the case, but didn't want to assume. But "grammar school"? US usage = elementary school? Gwinva (talk) 07:49, 26 September 2008 (UTC)[reply]
Synonyms in U.S. usage. Probably older folk use "grammar school", younger folk, "elementary". Of course now it's all hopelessly confused, with "middle schools" and such :) - Nunh-huh 07:53, 26 September 2008 (UTC)[reply]
Again, thank you! Where I come from, grammar schools are secondary schools (high schools)... As for packets, they're generally just called "hand outs" or "work sheets". A people divided by a common language, and all that... Gwinva (talk) 07:59, 26 September 2008 (UTC)[reply]

wikitext needs your help

About 2500 words have been written in response to the OP. Maybe some of them could go into Wikijunior:Solar_System? Saintrain (talk) 16:40, 26 September 2008 (UTC)[reply]

Does anybody know color of Neptune's sky. To me it looks ocean blue. If Neptune's sky is blue how blue. Is it dark blue?--57Freeways 00:16, 25 September 2008 (UTC)[reply]

I'm not really sure how you define "sky" for a gas giant. There is no real surface to stand on and look up - by the time you reach anything solid it will be dark (well, there will be no sunlight - it's actually white hot at those depths, I believe). The atmosphere above you will most likely look very different at difference altitudes. The Earth's sky is blue due to Rayleigh scattering, Neptune looks blue from the outside due to its absorption spectrum - those are very difference causes. I think the absorption spectrum would still make the sky appear blue from the inside, but I don't know what would happen with scattering, so it could be any colour. --Tango (talk) 01:58, 25 September 2008 (UTC)[reply]
I think Tango's caveats still apply. "Above the cloud tops" is a fairly vague definition for a gas giant. Neptune's Great Dark Spot, for instance, is thought to be a gap in cloud cover rather than a cloud itself. Additionally, "true color" is a very tricky concept when it comes to photography. Note NASA's photo gallery of Neptune, where Voyager's near-true-color image is darker and less saturated than the HST's. Images at SolarViews show the same contrast. — Lomn 15:38, 25 September 2008 (UTC)[reply]
Above the clouds? Wouldn't that be above the main atmosphere? If it is then the "sky" would appear black. ~AH1(TCU) 01:41, 26 September 2008 (UTC)[reply]

But the article said neptune's sky is blue. Uranus is too. neptune I thought has a haze layers which would put the sky color to be ocean blue. It wouldnot be black because it have an atmosphere. Uranus have a haze layers so it's sky is thought to be light blue, just like Earth's same as Saturn's.--57Freeways 22:36, 26 September 2008 (UTC)[reply]

Those are photos taken from the outside. "Sky" is what you would see looking up from inside the atmosphere, that's not necessarily the same colour as the atmosphere from the outside. --Tango (talk) 23:03, 26 September 2008 (UTC)[reply]

Global Warming Counterargument

I was debating this subject with someone, and he said that the temperature will not continue to rise indefinitely as proposed by "An Inconvenient Truth" (I'm not sure that Al Gore was making that point; it's what my opponent said) and that if one examines the history of Earth's climate, we are in a relatively stable period; essentially, stability is abnormal. Is there any truth to these claims?CalamusFortis 04:29, 25 September 2008 (UTC)[reply]

Scientists have been arguing over this topic for years, so unfortunately we can't provide THE answer. For starters, you might enjoy "The Great Global Warming Hoax?", which predicts global cooling. Also New Zealand Climate Science Coalition, which offers links to many interesting research articles. Gwinva (talk) 04:49, 25 September 2008 (UTC)[reply]
I don't think anyone claims the temperature will rise indefinitely. If they do, they are clearly wrong. There is only so much heat coming from the sun, so the planet will have to reach equilibrium eventually, whatever happens. --Tango (talk) 14:46, 25 September 2008 (UTC)[reply]

You should also know that many of the scientists who were quoted on the great global warming hoax consequently sued C4 for taking their remarks out of context...82.22.4.63 (talk) 17:39, 25 September 2008 (UTC)[reply]

Did they? Nice to know. I was merely offering links which might provide some of the "counterarguments" requested by the OP, but since I've not read either of them, I'm certainly not in a position to judge their academic rigour. Gwinva (talk) 02:41, 26 September 2008 (UTC)[reply]
Indeed. And neither Al Gore nor any scientist worth the appellation claims or claimed that temperature will rise "indefinitely". As a first, very rough approximation, it cannot become hotter than the maximum of its current core temperature and the surface of the sun without violating the second law of thermodynamics. More realistically, the current predictions center around about 3° Celsius temperature increase per doubling of atmospheric CO2. The normal difference between icehouse and hothouse modes of Earth's climate are about 10° degrees Celsius (globally - much more in high latitudes), so I would not consider a very much higher rise due to anthropogenic greenhouse gases likely. Note, however, that even that would have devastating effect - including complete meltdown of the ice caps, sea level rise of 60-70m (enough to flood most costal plains and cities), and corresponding social and political unrest (to put it mildly). On the plus side, all that will take a long while - IPCC projections for 2100 are "only" 9-88cm, and while thy are conservative, they are not outrageously so. And yes, while civilization will suffer and may even break down, life will almost certainly go on for several million years more. --Stephan Schulz (talk) 18:10, 25 September 2008 (UTC)[reply]

Certainly an "indefinite" rise is right out of the question...however, it seems technically possible that we could conceivably drive the planet to the point where it looks like Venus...a surface temperature of 460 degrees C. (Well, actually, the earth is further from the sun - so it would be a bit lower than that - but not by much). To get to that point, we'd have to initially drive the temperatures up to the point where deep ocean methane-hydrides would melt and dump methane (a nasty greenhouse gas) into the atmosphere and the oceans. That would cause plants and algae to die off on vast numbers - that would start to shut off CO2 absorption - CO2 from rotting vegetation, forest fires and volcanoes would build up in the atmosphere causing still higher temperatures - more plant die-off - less CO2 absorption - more heat...with nothing to stop it. Gradually, the increasing acidity of the atmosphere combined with higher temperatures would cause carbonate rocks to start to boil off CO2 - yet more greenhouse gasses - eventually, we'd hit 100 degC and then the oceans would boil - and the resulting enormous quantities of water vapor (which is an even bigger greenhouse gas than CO2) would drive things even further off the rails. As these additional gasses enter the atmosphere, it gets denser and denser - and that too pushes the heat up and promotes other chemical reactions to acidify the atmosphere. Eventually, thermodynamics demands that things level off - but by then, Earth is Venus-2.0! So I think if we ever let things get so bad that large scale plant die-offs started to happen then we'd be unable to prevent runaway heating of the planet...reaching maybe 200 to 300 degC (Venus is at 460degC...so it's not impossible).

But it's pretty academic because we'd all be dead LONG before that happened.

It seems very unlikely to me that things could possibly get that bad - but there are a lot of 'runaway' situations here and when you get "positive feedback" in a system, it can be impossible to stop. It's hard to know for sure though because some things (such as increased evaporation of the oceans) can both help and hinder the process. More evaporation means more greenhouse-gasses (water vapor) - but it also means more fluffy white clouds which reflect sunlight back out into space again. In those cases, it's incredibly hard to predict when the point comes where the increase in cloud cover balances the increase in greenhouse gasses.

But I certainly wouldn't want to exclude the possibility of Venus-like conditions arising from this situation.

SteveBaker (talk) 22:43, 25 September 2008 (UTC)[reply]

Hi. Although stability is rare, on a human history scale we have been enjoying this stability for a very long time. So, if a sudden change occured, it would seriously disrupt civilization, becuase all of modern civilization has developed during this stability. Quoting another Wikipedian, such a drastic temperature rise like the one predicted "would be seen as nothing short of apocalyptic". Thanks. ~AH1(TCU) 01:39, 26 September 2008 (UTC)[reply]
Although we can only hope that SteveBaker's apocalyptic scenario won't arise, there is a potential trigger to release those oceanic clathrates. Permafrost areas in the Arctic have unusually large stores of unprocessed carbon, i.e. plant matter that has not yet been processed by microbes. Warming could potentially release this carbon, most unpleasantly as methane. I can't find the disturbing reading on the "budgets" involved (the potential releases would dwarf total anthropogenic emissions), but I did find a few papers [12] [13]. Of course, like everything GW-related, there is intense controversy on this. However, if we do get caught in a feedback loop (and considering that GW is manifesting strongly in the Arctic, it's not impossible), that little bit of extra warming could be enough to release methane hydrates from the ocean on a large scale. Then we would have a problem! Franamax (talk) 03:18, 26 September 2008 (UTC)[reply]
What makes the "runaway to Venus" unlikely is that the temperature of Earth was quite a bit higher at some times in the past (due, among others, to changes in orbital cycles, continental drift, and tectonic processes affecting the atmosphere), and that we quite possibly already survived at least one Methane hydrate gasification - well, the lucky 10% or so of species did. Of course, this may well be a stochastic process, and we got lucky. We may or may not get lucky the next time (if any), especially considering that the sun is (very slowly) warming up... --Stephan Schulz (talk) 19:26, 26 September 2008 (UTC)[reply]

Chemistry

What is the basic Difference between Radical and Ion.I Think that Radical is the Group of charge molecule. EG NH2+ please satisfy me.i will be very thankful to u. —Preceding unsigned comment added by 119.152.246.120 (talk) 05:24, 25 September 2008 (UTC)[reply]

A radical is a molecule, atom, or ion with an unpaired electron. An ion is a molecule or atom with a different number of protons than electrons. An ion may be a radical, but there are ions that are not radicals, and radicals that are not ions. Someguy1221 (talk) 05:39, 25 September 2008 (UTC)[reply]

Histogram of distribution of skin color

I had an odd thought last night: what would an accurate histogram showing the distribution of skin color in the world look like? Anybody have a guess, or the data needed to put one together? --98.217.8.46 (talk) 12:28, 25 September 2008 (UTC)[reply]

Von Luschan's chromatic scale has a map showing global distribution according to his classification, however it doesn't deal with respective populations of the area. -- MacAddct1984 (talk &#149; contribs) 14:02, 25 September 2008 (UTC)[reply]
Right, I'm interested in populations. But I'm not interested in imagined "native populations", either, I'm interested in the current population. --98.217.8.46 (talk) 14:48, 25 September 2008 (UTC)[reply]

2D PAGE not sensitive to low-abundance proteins

I'm reading a journal article about a 2D PAGE which did not show some of the proteins expected "possibly due to their low abundance". They then say "more comprehensive analyses will be performed to identify the altered low-abundance cellular factors by using large-size gel slabs, depletion high-abundance proteins, enrichment of the samples by pre-fractionation and by using silver staining coupled with LC-MS/MS analysis". How do you suppose they might deplete high-abundance proteins, leaving the low abundance ones unaffected? What do they mean by pre-fractionation? Is silver-staining more sensitive or does it just enable the LC-MS? --Seans Potato Business 13:21, 25 September 2008 (UTC)[reply]

Using high performance liquid chromatography and liquid chromatography-mass spectrometry, i.e. LC-MS. See this reference. Silver staining assists PAGE. See this reference. Axl ¤ [Talk] 16:15, 25 September 2008 (UTC)[reply]
I have an article on my desk I've intended to read for months now. "Recent Developments in High-Abundance Protein Removal Techniques" in LCGC North America vol 26 no 3 p 278 by Tim Wehr. My quick skim of the article suggests it is exactly what you are asking about. I think that articles from this non-peer review mag are available on line for free. ike9898 (talk) 16:42, 26 September 2008 (UTC)[reply]
Hey, thanks for the tip. A quote from the arrticle: Using "split-couple-recombine" combinatorial systhesis, a library of hexapeptides bound to macroporous 65-μm-diameter polymethacrylate microbeads is produced, with each bead carrying a unique peptide sequence. The sequential coupling of the 20 protein amino acids results in a library of 206 unique beads. These are packed in spin columns with a 100-μL bed volume. In contrast to antibody-based immunodepletion products, which typically have a capacity of 100 μL or less, sample volumes of 1 mL and more can be applied to the ProteoMiner beads. Because there are a limited number of binding sites for each protein, HAPs quickly reach the bead capacity, and excess HAPs are passed through the column. After sample introduction and washing, bound proteins are eluted from the column with a small volume (≤300 μL) of elution buffer (5% acetic acid, 8 M urea, 2% CHAPS). The net result is depletion of HAPs accompanied by at least a threefold increase in the concentration of LAPs - is this not a ridiculous idea? Surely some LAPs (low abundance proteins) simply wont bind to a hexapeptides and be flushed staight through? I see no reason to assume all or even most LAPs would recognise any hexapeptide. --Seans Potato Business 22:42, 27 September 2008 (UTC)[reply]

physical draw backs of jogging

1.does jogging makes a man slim quickly but as we leave the routine ,fatness reverses fast again. 2.does it have any drawbacks for jogging after some eating. 116.71.188.65 (talk) 14:26, 25 September 2008 (UTC)[reply]

Moved poster's sig to proper spot. Franamax (talk) 14:34, 25 September 2008 (UTC)[reply]
We can't give medical advice. Ask a doctor/nutritionist/personal trainer. --Tango (talk) 14:42, 25 September 2008 (UTC)[reply]
This isn't medical advice...1) Jogging, like any physical activity, will help to burn calories - that's not guaranteed to cause weight-loss but increased exercise is reasonably well correlated with becoming 'healthy' (so if you are overweight it may burn off fat). Obviously if you stop jogging then your calorie-burning will reduce (unless you replace the activity). To combat this you could take in less calories, or burn calories through other activities. 2) Jogging after eating might be uncomfortable if you feel 'full up' so aren't as flexible. Similarly running on an empty stomach could cause discomfort too. 194.221.133.226 (talk) 14:52, 25 September 2008 (UTC)[reply]
The first question might not be, but the second is. If we say there are no drawbacks and then the OP goes jogging after eating and falls down ill because of it, we could end up in a lot of trouble. --Tango (talk) 15:00, 25 September 2008 (UTC)[reply]
No, we wouldn't end up in trouble. The no medical advice bit is not a legal requirement. In general, there's no reason not to jog after eating, it is up to your own preference. Some people are fine with it, others feel uncomfortable. This is not medical advice tailored to your situation, this is just a general statement of pretty commonly known information. --98.217.8.46 (talk) 22:59, 25 September 2008 (UTC)[reply]
I believe the Wikipedia:Medical disclaimer prevents us from getting into legal trouble. We don't give medical advice here to protect OPs from harm. Zain Ebrahim (talk) 12:42, 27 September 2008 (UTC)[reply]
I agree with anonymous editor 194.221.133.226. This is not "medical advice". A doctor or nutritionist would be no better informed than anyone else (although a personal trainer might be). As 98.217.8.46 states, the answer to question 2 is derived from common sense and general knowledge. Axl ¤ [Talk] 07:39, 26 September 2008 (UTC)[reply]

How is water taken up by the body?

I feel silly for asking, but I seriously can't find any good info. I guess my search terms are pretty bad. When drinking water, where in the body is it taken up? Does stomach acid matter at all? Does having a full/empty stomach matter? And, perhaps most importantly, how long does it take to clear out of the stomach? Thanks in advance. -- Aeluwas (talk) 14:39, 25 September 2008 (UTC)[reply]

It's absorbed in the intestines, both small and large, according to those articles. I don't know about the rest of your questions. --Tango (talk) 14:58, 25 September 2008 (UTC)[reply]
From Guyton's Textbook of Medical Physiology, 11th edition, chapter 65. Very little water is absorbed in the stomach. This is because the internal lining (epithelium) lacks villi, which are abundant in the small intestine. Between six and eight litres of water are absorbed from the small intestine each day. A further one and a half litres are absorbed in the colon. [Most people drink between one and three litres each day. The "extra" water absorbed comes from secretions into the gastrointestinal tract.] Stomach acid makes no difference. Axl ¤ [Talk] 16:50, 25 September 2008 (UTC)[reply]
Thanks guys. I also found these links: gastric emptying and the effect of omeprazole on water gastric emptying. It looks like ~15 minutes would be a normal time to get rid of 50% of the water. -- Aeluwas (talk) 19:55, 25 September 2008 (UTC)[reply]

Hair Growth in Mammals

in many cows ,dogs etc the hair cant grow from certain point but coats ,man etc it grows continously .can some explain me this fact. —Preceding unsigned comment added by 119.154.2.133 (talk) 16:20, 25 September 2008 (UTC)[reply]

Hair follicle goes through cycles. At the end of the cycle the hair is shed. Some animals (cows) have shorter cycles so the hair stays short. Others (people) have longer cycles. A person's eyebrow's follicles have shorter cycles (a few months) than head hair follicles ( a few years) so head hair can grow much longer before it sheds but eyebrows stay short. Saintrain (talk) 19:23, 25 September 2008 (UTC)[reply]
But then why would human head hair have such long cycles? --Kjoonlee 23:46, 26 September 2008 (UTC)[reply]

Are cows immune to CS gas?

Just heard someone on a radio phone-in quote the factoid that cows are supposedly immune to the effects of CS gas. Anyone able to confirm/deny? It doesn't sound to me like something that anyone would even bother to test... --Kurt Shaped Box (talk) 16:57, 25 September 2008 (UTC)[reply]

CS gas does not address cows specifically, but says Notably, CS has a limited effect on animals due to "under-developed tear-ducts and protection by fur". -- Coneslayer (talk) 17:03, 25 September 2008 (UTC)[reply]
I'm sure they would bother to test it, since virtually all chemical weapons (even mild ones like this) are tested extensively on animals. --Sean 18:57, 25 September 2008 (UTC)[reply]
But I don't know if cows are a common test subject Nil Einne (talk) 20:33, 25 September 2008 (UTC)[reply]
I've been pondering this on-and-off since I posted the question. Perhaps a herd of cows was used at some point as an experimental substitute for a crowd of hostile rioters when testing CS gas grenades in the open air? Only thing I can think of (though it doesn't really sound plausible)... --Kurt Shaped Box (talk) 22:11, 25 September 2008 (UTC)[reply]
Is there a plausible situation in which the use of CS gas against protesters also affected a herd of cows? It could have been an incidental observation. Plasticup T/C 22:31, 25 September 2008 (UTC)[reply]
These sorts of factoids are often over-specific to make them seem more obscure. (ie : "It's illegal to hitch a pet [exotic animal] to a fire hydrant.") Perhaps someone took the fact that animals are resistant to CS gas and decide to restate it as "cows are resistant". It wouldn't take too many retelling for that to become "cows are immune". APL (talk) 01:07, 26 September 2008 (UTC)[reply]
Cows have big eyes as well as noses with mucous membranes, neither protected by fur. I doubt that a cow in a cloud of CS would be immune to its effects. As soon as the irritant contacts the eye, the interior of the nose, the mouth and the lungs it has its effect. Having fur might protect somewhat against direct skin contact, which is the least of your worries. Edison (talk)

Why do people prefer creamy skin lotions compared to solid or liquid formulas?

I got addicted watching skin cream ads on YouTube and wondered why every lotion and cream for the skin is a whipped white concoction? Is there something humans like about white creamy lotions as opposed to colored oily or watery formulas? Do any wild animals put anything on their skin to soothe dryness? --70.167.58.6 (talk) 22:19, 25 September 2008 (UTC)[reply]

Any coloring agent in the lotion is likely to stain skin and clothing. --Sean 22:54, 25 September 2008 (UTC)[reply]
Pigs roll in the mud to cool themselves off... I don't know if they get any moisturizing effect out of it to (or get one intentionally, anyway). --98.217.8.46 (talk) 23:03, 25 September 2008 (UTC)[reply]

With a creamy lotion, it's easier to control where you're applying it than if it's more watery. --Anonymous, 03:03 UTC, September 25, 2008.

What about it was gelatinous or paste? --70.167.58.6 (talk) 16:30, 26 September 2008 (UTC)[reply]
Well, people certainly use petroleum jelly (a clear jelly/paste) as a moisturizer. (The original Vaseline was petroleum jelly.) Although it works (sometimes better than other moisturizers), petroleum jelly is usually not preferred anymore because of the "greasy feel". If you listen to the adverts, you'll see they'll often mention "it doesn't feel greasy!" Also being noncomedogenic (not clogging pores/forming acne) is important for some people - petroleum jelly may cause acne issues, depending on how/where it's used. The greasy/non-greasy issue is also probably why medicinal creams are preferred these days to ointments and liniments -- 128.104.112.147 (talk) 19:41, 26 September 2008 (UTC)[reply]
So what makes "greasy" skin cream less undesirable? What's the psychology behind that? --70.167.58.6 (talk) 14:50, 29 September 2008 (UTC)[reply]
I don't know exactly, but I'd guess it's due to discomfort with there being "something" on your skin, or concern over transferring the grease elsewhere. Ointments/moisturizers are usually applied with the hands, and once finished applying, you usually have to clean up your hands with a "greasy" product (or else there is still the "grease" on your hands), while non-greasy products usually absorb into the skin well enough not to require cleanup. -- 128.104.112.147 (talk) 23:18, 30 September 2008 (UTC)[reply]

QFT Example?

The pages under quantum field theory seem to lack examples. Could someone show an example of, say, an electron in a box, perhaps radiating photons to go to lower states, using a QFT, so those of us who aren't intimately involved in it can appreciate better how it works? --Zemylat 22:44, 25 September 2008 (UTC)[reply]

September 26

"hurricane" in France

Hi. I think I read somewhere that a "hurricane", most likely an extratropical cyclone, struck France causing $88 billion in damage sometime in the late 20th century, second costliest worldwide only to Katrina. Also, is there an article on this? The global tropical cyclones track image, a featured picture, which does not distinguish between extratropical and tropical systems, shows at least one category one hurricane approaching France. Also, is there an article for the hurricane-strength cyclones that struck Britain I think in 1990 (and killed Emily Macdonald), and recieved "names"? Also, does anyone know is there an online database where I can search, for example, the last time (year, name) that a tropical cyclone of x intensity approached within x km of a specified location, etc? Thanks. ~AH1(TCU) 01:54, 26 September 2008 (UTC)[reply]

Wikipedia has a pretty extensive catalog of tropical cyclones:
-- MacAddct1984 (talk &#149; contribs) 02:17, 26 September 2008 (UTC)[reply]
The French one was in December 99, just 1 or 2 days after Christmas. I cannot remember the name of the cyclone, though, sorry --Lgriot (talk) 02:28, 26 September 2008 (UTC)[reply]
I found it!: "The second storm event occurred on December 26. It was caused by a low named "Lothar", and left a trail of destruction from northwestern France to southern Germany and Switzerland." full text at: [14] . We also have an article: Lothar_(Storm) --Lgriot (talk) 02:37, 26 September 2008 (UTC)[reply]
Hurricanes have to be tropical cyclones (or sub-tropical cyclones, as the definition was expanded recently). Extratropical cyclones, not matter how strong, cannot be hurricanes. See Cyclone, specifically Extratropical cyclone, Subtropical cyclone, and Tropical cyclone. Plasticup T/C 03:35, 26 September 2008 (UTC)[reply]
Following the external links in the Lothar article, I find that it caused 5.0 billion

Euros damage - less than the 1990 storms which hit France. So the "second costliest worldwide only to Katrina" part cannot be correct. Hurricane Katrina caused about $100 billion damage. Rmhermen (talk) 13:25, 27 September 2008 (UTC)[reply]

Hi. Yes, but farther down in the pdf it states that total damage was over 100 billion euros. Also, is there an article for the 1990 storms? What about the database? Thanks. ~AH1(TCU) 19:54, 27 September 2008 (UTC)[reply]

ACCA

Hello I want the free Notes for ACCA if some help me i will be very thenkfulPDF). Also what is the salary given to ACCA with one year experience in UK(in digits please) —Preceding unsigned comment added by 119.154.29.138 (talk) 02:39, 26 September 2008 (UTC)[reply]

Association of Chartered Certified Accountants and Chartered Certified Accountant may be of some help. SteveBaker (talk) 03:42, 26 September 2008 (UTC)[reply]

Kinetic Energy of Helium

Yup, I'm doing homework. I thought I'd done the question right, but can't get my online homework board to accept my answer. To find the total kinetic energy of 2.4 kg He at 298.15 K, I used the formula KE (avg) = (3/2)RT, then multiplied this by the number of moles.
So: (3/2)(8.314 J/mol*K)(298.15K) = 3718.2 J/mol
(2.4 kg He)(1000g He/1 kg He)(1 mol He/4.003 g He) = 599.6 mol He
(599.6 mol He)(3718.2 J/mol He) = 2.229E6 J

There is a second part to this problem, but I thought I'd see if I was doing something incorrectly in this first part, or if I am at least right to this point? Any pointers to my approach and setup of this problem would be appreciated. 166.70.30.42 (talk) 07:19, 26 September 2008 (UTC)Andy[reply]
Nevermind...I figured it out. Damned significant figures get me every time... 166.70.30.42 (talk) 07:40, 26 September 2008 (UTC)Andy[reply]

bird nomenclature / indie rock lyric question

Joanna Newsom is a (really hot) singer who writes rambling 11-minute songs bursting with precious avian references. One memorable opus begins, "The meadowlark and the chim-choo-ree and the sparrow set to the sky in a flying spree, for the sport over the pharaoh". What the heck is a chim-choo-ree? I searched the encyclopedia for a bird by that name but found only the Mary Poppins song.--The Fat Man Who Never Came Back (talk) 07:52, 26 September 2008 (UTC)[reply]

ArthurMag has an article on Joanna Newsom where it is suggested that she probably derived "chim-choo-ree" from Mary Poppins. --Bruce1eetalk 14:13, 26 September 2008 (UTC)[reply]
Thanks. That's a good reference--funny too: "Meadowlarks and sparrows are songbirds, of course, but the chim-choo-ree, as far as I can tell, is probably an outgrowth of watching Mary Poppins too many times." But isn't the song "Chim Chim Cher-ee" about chimneys and not about birds? Ms. Newsom is an odd duck. Anyway, sorry to bother the Science desk with this pop culture rubbish; I imagined that a chim-choo-ree was some sort of archaic slang for a specific songbird.--The Fat Man Who Never Came Back (talk) 16:03, 26 September 2008 (UTC)[reply]

Severity of Decelleration Required to Cause Traumatic Brain Injury

I've seen that rapid decceleration can cause diffuse brain injury in numerous places, and that this can occur/ is likely to occur in auto accidents. However, I have never seen it mentioned exactly how severe the accident needs to be, nor the neccesary ammount of decceleration. So, this is my question: what level of acceleration is neccesary to cause brain injury, is coming to a quick stop without impact in a motor vehicle capable of causing brain injury, and (pending upon the answers to the first two) how is it that proffessional race car drivers/astronauts/sky divers/etc. do not sustain injury for these reasons? Thank you in advance for any help. Phoenix1177 (talk) 13:35, 26 September 2008 (UTC)[reply]

Hmm, well internal bleeding is common from deceleration in high speed accidents. Since this can injure your organs, this would presumably apply to your brain as well. I would assume it doesn't affect (or is it effect?) the people you listed because they don't actually decelerate all that quickly. If a professional driver were to slam into a wall and go from 150mph to 0 in no seconds flat, internal bleeding would probably be a major concern (or the least of their worries?). -- MacAddct1984 (talk &#149; contribs) 13:54, 26 September 2008 (UTC)[reply]
Thank you for your reply. A large part of my question stems from the fact that someone wrecking into a wall at 60 mph has a moderate chance of brain injury, whilst it doesn't seem common sensical that someone going 60 mph and slamming on their brakes would have equal chances(mainly because people slam on their brakes a good deal and don't seem to sustain such injury) So, I figure that either those extra few miliseconds of stopping time and meters of distance with the brakes are a mitigating factor, or that most people involved in high speed action should be at high risk for brain injury. [the case for sky divers is probably unreasonable, but with race car drivers it would seem somewhat more logical] Phoenix1177 (talk) 14:07, 26 September 2008 (UTC)[reply]
If you're not in a race car, your braking and turning maneuvers won't be much higher than 1g. -- Coneslayer (talk) 14:27, 26 September 2008 (UTC)[reply]
If you hit the brakes full on at any speed the reduction in speed will be not even in the same league as a crash. If i'm correct I think a big thing is the 'g-force' you go under in a crash. There is G-force#Human_tolerance_to_g-force that may be of interest. In particular the 'involuntary' section showing crashes. A vehicle braking will get some g but it won't be anywhere near that of a crash. The more 'time' that can be introduced into the deccelaration the better, that's the purpose of crumple zones and airbags and etc. etc. 194.221.133.226 (talk) 14:21, 26 September 2008 (UTC)[reply]
This study demonstrated deformation of brain tissue during deceleration of 20–30 m/s2, with the neck unprotected. Allowing the neck to pivot freely increases the strain just above the base of the brain where the brain is mostly strongly adherent to the surrounding bone. Professional racing drivers and astronauts wear neck protection to prevent whiplash injuries. This also helps to reduce torsion of the brain at its base. Axl ¤ [Talk] 14:34, 26 September 2008 (UTC)[reply]
More specifically, a basilar skull fracture is the danger when your body stops faster than your head does. The bone sutures can split and tear open the sacs holding in your brain's precious bodily fluids. --Sean 16:54, 26 September 2008 (UTC)[reply]
You also have to worry about your head hitting the windshield/steering wheel and your chest hitting the steering column (wear your seatbelts!). Although, I did go on a call to a car accident where the patient had a collapsed lung, even though the airbag deployed... -- MacAddct1984 (talk &#149; contribs) 17:45, 26 September 2008 (UTC)[reply]

Plant identification questions (yew and juniper)

I would like some help identifying some plants. I have some yew shrubbery and a juniper tree in my yard. I would like to know more specifically what type of yew and juniper I have. I live in Richmond, Virginia. The juniper is about 15 feet tall, and in a general cone shape. My brother believes it is Juniperus communis, but we weren't entirely sure how that differed from Juniperus virginiana. My concern is the berries. The berries are not round, but instead are sort of spikey. What sort of things can I look at to identify the type of juniper? Any details I could provide online that could help someone identify it? As for the yew, my brother believes it is a Taxus cuspidata, but I wasn't sure how that differed from Taxus baccata. Again, what sort of things should I look for to tell what type of yew it is?

I could provide photos if necessary, but I'd also like to learn what to look for myself. Thanks!-Andrew c [talk] 16:18, 26 September 2008 (UTC)[reply]

Evolution of axon membranes

We were studying the membranes of nerve axons today in biology and it confused me as to how such a system could evolve? We were studying different channel proteins and carrier proteins etc. Anyway, wouldn't any organism without such pumps like sodium-potassium pumps die? How else could organisms without proteins to carry out active transport survive? It seems unlikely that an entire membrane could evolve in one generation of a cell. I'm not trying to encourage creationism or anything, I do believe in evolution, it's just a bit confusing.

As a second kind of follow on question, are there any organisms that survive or is it at all possible for an organism to form without adenosine triphosphate mechanisms? Thanks a lot :)

Cyclonenim (talk · contribs · email) 17:18, 26 September 2008 (UTC)[reply]

"It seems unlikely that an entire membrane could evolve in one generation of a cell" - that's correct and as sure as I am that my username is Seans_Potato_Business, I don't believe it evolved over the generation of a single cell. There's not telling how each of the proteins involved (you can look for clues by checking sequence homology among genes and among organisms) but each gene and gene-product may have started off as something else. It's nor necessary (nor possible?) to known or understand each evolutionary step which occured. ----Seans Potato Business 17:36, 26 September 2008 (UTC)[reply]
I do appreciate that, it's not really a simple discussion. My point was, however, that wouldn't an organism die without just one type of these intrinsic proteins? If it didn't evolve all at once, how could it come into existence at all? —Cyclonenim (talk · contribs · email) 18:10, 26 September 2008 (UTC)[reply]
All organisms require ATP or GTP via biochemical respiration. However this paper suggests that bacterial motility may occur through a different intermediate in oxidative phosphorylation. Axl ¤ [Talk] 18:11, 26 September 2008 (UTC)[reply]
A modern organism might die without one protein from the pool. However this is no reason to suppose that an evolutionary ancestor would die without that protein. Axl ¤ [Talk] 18:14, 26 September 2008 (UTC)[reply]
It is noted that a wide range of living things have similar needs when it comes intracellular K and Na levels. As you are apparently are aware, much modern life maintains the right ranges by actively pumping Na and K against the natural gradient. The easiest explanation for how that situation might have evolved is if there was some very primitive progenitor that lived in an environment that didn't need pumping, i.e. to assume that once upon a time the external Na and K levels matched the internal levels we have today. The evolution of active pumping would allow a cell to escape that environment, but the existence of active pumping wouldn't be necessary to live there. Though I don't think it tells the whole story, the chemistry of the oceans themselves were radically different early in life's history (high Fe(II), lower Na, negligible O), so it certainly plausible that life initially began under very different environmental constraints than exist today. Dragons flight (talk) 18:26, 26 September 2008 (UTC)[reply]
Another way to look at it is this: It is possible that a proto-organism lived at equilibrium with the salt water environment, but that evolution of channels capable of separating charge across the membrane would be extraordinarily beneficial and allow the descendents of that proto-organism to thrive. The neuronal axon is an extremely specialized situation that has evolved over many millions of years. However, other cell types in the body also use gradients for transporting all sorts of things across the membrane, which you might imagine would be advantageous to a single celled organism in the primordial soup. Also, the ability to build up charge on the surface and discharge it upon some stimulus could be considered as a possible form of primordial warfare. There are plenty of explanations for how a complex adaptation like the neuronal cell membrane could evolve in small steps, each of which could be advantageous to the given organism in its particular context, without calling forth the "irreducible complexity" argument (i.e. "It seems unlikely that an entire membrane could evolve in one generation of a cell.") favored by the intelligent design crowd. By the way, you don't need to "believe" in evolution. You just have to understand how it is actually theorized to work, think up some testable hypotheses, and look for ways to confirm or refute them! Have fun. Medical geneticist (talk) 22:45, 26 September 2008 (UTC)[reply]

Making gold

Is it theoretically possible to make gold out of common atoms like hydrogen or carbon? Is there any serious scientific investigation going on in this field?Mr.K. (talk) 18:27, 26 September 2008 (UTC)[reply]

See Synthesis of noble metals#Gold -- MacAddct1984 (talk &#149; contribs) 18:35, 26 September 2008 (UTC)[reply]
Theoretically all heavy elements are derived from lighter elements (see Supernova nucleosynthesis). I'm not sure if something as light as hydrogen or carbon could be made into something as heavy as gold in a meaningful way with current technology (keep in mind the links above are just about going from specifically Mercury to Gold, which is just a proton difference). But theoretically it's possible—esp. if you were using fusion technology. But it's not likely ever a worthwhile replacement to mining. --98.217.8.46 (talk) 22:13, 26 September 2008 (UTC)[reply]
I can't help but wonder: what worth would gold have if you could make it in your cellar? hydnjo talk 23:22, 26 September 2008 (UTC)[reply]
It would still be useful for electrical contacts and it would still look pretty so would be used for cheap jewellery, so it would have some value. It would lose most of its value, though. --Tango (talk) 23:30, 26 September 2008 (UTC)[reply]
For sure gold has practical value. I was just musing about the non-practical value should gold become abundantly available as was suggested by Mr.K. There would be some advantage I'm sure in using gold foil in place of aluminum foil for some everyday applications ;) hydnjo talk 01:36, 27 September 2008 (UTC)[reply]
Beisides that, hydnjo, if I could make in my cellar that would not disturb the market price of gold. It would only improve my financial situation. Mr.K. (talk) 10:54, 27 September 2008 (UTC)[reply]
C'mon now Mr.K., surely you'd share your recipe with us and you know what blabber mouths we are. -hydnjo talk 12:33, 27 September 2008 (UTC)[reply]
The amusing thing is, Mr. K, if you had the technology to make gold out of carbon, you'd make more money licensing that than you'd ever make by producing gold. --98.217.8.46 (talk) 14:34, 27 September 2008 (UTC)[reply]
By selling the gold you make in your cellar you would depress the price of gold - it's simple supply and demand. If you were disciplined and only made a small amount of gold, the affect would be negligible, but if you tried to make massive amounts (something in the region of hundreds of tonnes, I'd expect) you could cause a massive crash in the price of gold. --Tango (talk) 15:22, 27 September 2008 (UTC)[reply]
I think he means if everyone could make gold in their cellars. — DanielLC 16:40, 27 September 2008 (UTC)[reply]
Also, that scenario reminds me of an episode of The Twilight Zone (specifically, the end). — DanielLC 16:42, 27 September 2008 (UTC)[reply]

Extremely deadly tranquillisers?

I was watching The Lost World and one of the characters mentioned that if someone accidentally shot themselves in the foot with one of their tranqs, it would kill them instantly. Are there any real tranqs that are this deadly, where a single shot to the foot would instantly kill a man? ScienceApe (talk) 20:04, 26 September 2008 (UTC)[reply]

Tranquillisers don't take effect instantly. They take seconds or minutes to take effect. The dose of tranquilliser determines the risk of death. If the tranquilliser is intended to sedate an elephant, it could well kill a person. Axl ¤ [Talk] 20:22, 26 September 2008 (UTC)[reply]

Tango, so you siad the sky color is not always the atmosphere on the top?? Well, Venus sky is orange-scarlet or coral seen from the surface because the orange-rd smog. Wouldn't the middle of cloud layers be yellow or yellow-orange. Does anybody know what's the color of Venus' sky above the cloud-tops, because Venus have 3 main layers of clouds.--57Freeways 23:20, 26 September 2008 (UTC)[reply]

A good example of a planet with a different coloured sky from the inside than the atmosphere appears from the outside is Earth - the sky is blue but seen from space the atmosphere is pretty much colourless. I don't know about Venus, I'm afraid. --Tango (talk) 23:27, 26 September 2008 (UTC)[reply]
  • Above the clouds is probably above the main atmospher of Venus. If so, then it's probably black. It's uppermost layers is probably acid clouds. Our sky is light blue over Trophospher', the next layer of atmos' is probably dark blue, the toppest layer of our sky might be indigo.--57Freeways 00:03, 27 September 2008 (UTC)[reply]

September 27

Where does my power come from?

I know it comes from a power plant. I live in West Virginia, US. We have a coal fired power plant less than 50 miles from where I live. Someone told me that the energy produced there is sent the the western US and we get our power from somewhere else. Is there anyway to figure out what specific power plant provides the energy for my area? —Preceding unsigned comment added by 96.240.72.130 (talk) 01:35, 27 September 2008 (UTC)[reply]

See Electric power transmission. As I understand it, the power generated at your local station is put into "the grid," which can be seen as a collective pool that consumers draw power from. So you don't actually get power from any particular power plant. It's possible that the electricity produced at your coal plant is being sold to cities in the West, and that your town is buying power from some other source, but that doesn't mean these plants are literally shipping their electricity to the buyers. It's all just going onto the grid. --Allen (talk) 03:09, 27 September 2008 (UTC)[reply]
And looking at the US grid map, WV is all in the East Central region of the Eastern Interconnect, so it is indeed the same grid. It is quite possible that the wires coming from the coal plant run in a westerly direction, but it seems unlikely that the power is being marketed into the Western Interconnect. Franamax (talk) 16:11, 27 September 2008 (UTC)[reply]
The power systems analyze flows on the grid by instruments located at substations, generating stations and on trnsmission lines. The amount flowing from each source and to each load varies depending on contidions. Computer modelling can determine the effect of adding more or less power to the system at any one node. If the effect of turning your local power plant on or off were compared to the flow to a load in the western US, I expect that lettle effect would be seen, In other words, it would be more likely that your local plant is not literally supplying western load. For one thing, there would be considerable losses of power along the way due to resistance. Edison (talk) 20:04, 27 September 2008 (UTC)[reply]

would a free falling object hit with the same force from 40' vs. 20'

my sons cubs scouts is having a experiment with an egg wrapped in a package. it will be dropped from 40' to see if it breaks. My egg was ok at 20' but i have no way of testing it from 40'. I was just wondering if it would hit with the same force at 20' as it would 40'209.244.187.94 (talk) 04:17, 27 September 2008 (UTC) Thank you Ray B.[reply]

Depends on the package. See Terminal velocity. If the package has a design that maximizes air resistance (like a parachute, for example), then it might hit with the same force at 20' or 40'. Otherwise, it will hit with more force. You might try testing by standing at 20' and throwing the package downward, trying to make it leave your hand at roughly the same speed it hits the ground at when dropped from 20'. --Allen (talk) 04:26, 27 September 2008 (UTC)[reply]
Just throw the egg up 20' from where you were standing before and let it fall from there. — DanielLC 16:21, 27 September 2008 (UTC)[reply]
That would have no effective difference from droping it at 20'. If the egg is at the top of the throw at 20', it is at zero velocity. From there, the downward flights are identical. The idea behind the egg drop is to make something that will reach a relatively slow terminal velocity as soon as possible. Thus, if it hits that terminal velocity at 15', it will drop at that same speed whether you drop it at 20', 50', or 1000'. That's the idea behind terminal velocity. --Jayron32.talk.contribs 20:40, 27 September 2008 (UTC)[reply]
I think you misunderstand. I believe DanielLC's plan was to stand 20' above the ground and then throw the egg 20' in the air, so it is momentarily at rest 40' above the ground, so it will follow the same path as if it were dropped from 40'. --Tango (talk) 20:51, 27 September 2008 (UTC)[reply]

Sounds like a cool experiment. What sort of package did you and your kid design? Plasticup T/C 05:05, 28 September 2008 (UTC)[reply]

The pedantic answer is that terminal velocity is asymptotic. So the egg will not ever hit at exactly the same force.--Fangz (talk) 12:53, 28 September 2008 (UTC)[reply]
I think random fluctuations probably have more of an effect than it not having quite reached terminal velocity. --Tango (talk) 20:57, 28 September 2008 (UTC)[reply]

Penrose Diagram

I have no idea how to interpret this. Could someone please help me out? Also, what is a "lightlike infinity"?

Ζρς ι'β' ¡hábleme! 05:57, 27 September 2008 (UTC)[reply]

I resized the image, hope you don't mind. It was a bit... overwhelming before. Anyway, this diagram just shows ordinary flat spacetime in an unusual coordinate system given by tan (u±v) = x±t, or more explicitly by
u = (tan−1 (t+x) − tan−1 (t−x)) / 2
v = (tan−1 (t+x) + tan−1 (t−x)) / 2
The axes marked "space" and "time" are actually u and v respectively. This coordinate transformation maps the whole of spacetime into the diamond. The roughly horizontal and vertical curves are curves of constant t and x. Lines of constant x±t map to lines of constant u±v, so light still travels along 45° diagonal lines in this diagram. Its most useful property is that worldlines which diverge to infinity in the usual coordinates converge to the boundary of the diamond in (u,v) coordinates, so this diagram shows you the mathematical structure of spacetime at infinity. Timelike geodesics extend from the bottom point of the diamond (called "past timelike infinity") to the point marked "timelike infinity" (normally "future timelike infinity"). Lightlike/null worldlines extend from past to future lightlike/null infinity (lower and upper edges of the diamond). You can extend this to n+1 dimensions by using polar coordinates and transforming (t,r) as above while leaving the angles alone. The diamond generalizes to two (hyper)cones with apexes at past and future timelike infinity (which are still points) and glued together at spacelike infinity (which is a (hyper)sphere). (edit to add: Sorry, I think that's wrong; that mapping isn't conformal. The correct generalization, if I'm not still confused, is to imagine wrapping the diamond around a cylinder so that the left and right vertices (spatial infinity) coincide, roughly as shown here, and then generalize that to a higher-dimensional cylinder Sn × R. Spatial infinity is a point, not a hypersphere.) -- BenRG (talk) 12:00, 27 September 2008 (UTC)[reply]
That sounds good. They're like those Escher Circle Limit pictures but putting the whole of a 2D space-time in a small diagram. I must admit I always though Penrose diagrams was the name of a much more useful thing which in wiki is called Penrose graphical notation. Dmcq (talk) 12:11, 27 September 2008 (UTC)[reply]
Yes, Escher's Circle Limit pictures are based on the Poincaré disk model of hyperbolic space, which is a very similar idea. Also very similar is the Riemann sphere. They're all conformal maps. -- BenRG (talk) 15:16, 27 September 2008 (UTC)[reply]

Changes in mental processes due to multitasking

I posted this a couple of days ago on the Misc desk as sex differences in internet use and it was suggested I post here. I rewrite slightly:

It is a commonplace observation that women are able to multitask and men cannot. (I should clarify that "multitasking" can involve more than paying attention to electronic information. A sympathetic female journalist was commenting on the recent resignation of Ruth Kelly from the British Cabinet, "to spend more time with her family". The journalist said that once she had interviewed Kelly by phone, and after a few minutes had figured out -- and the politician had admitted -- that the splashing sounds in the background were of the two year old being bathed. Good thing the phone didn't drop in the water.)

It is a commonplace observation that those individuals most immersed in computers and the internet are male. Obviously these are both generalisations. One social change commented upon over the last ten years or so is the impact of prolonged and submersive internet use on our thinking and interaction patterns. (See for example the recent Financial Times anecdotal article and interviews here.) One theme is that our attention is being fractured; some observers see this as wholy negative, while others see more positive possibilities.

So we have two conflicting ideas: that the so-called male brain can only cope with one information stream at a time, and that many men and boys thrive in an atmosphere of information bombardment (multiple screens on computers, phones, TVs simultaneously). My question is, is there any serious research on this? And how are these conflicting tendencies reconciled? BrainyBabe (talk) 12:14, 27 September 2008 (UTC)[reply]

Whatever about commonplace observance the latest statistics as best as such things can be done for this say that more women than men use the internet and use it if anything a bit longer. Just google on 'percenage male female internet usage' and look for 2007 or later. As to multitasking it is something to be avoided if possible even for simple tasks with little overlap if you want to do them properly. Anybody driving round talking on their mobile is a menace. Dmcq (talk) 13:02, 27 September 2008 (UTC)[reply]
Or driving around while fixing their makeup. :) Zain Ebrahim (talk) 13:04, 27 September 2008 (UTC)[reply]
Yes, I think it's the assumption that multitasking is an amazing skill that is the problem with the whole thing. People assume that being able to do 5 things at once means you get 5 times as many thing done (and done just as well). You don't. Totally my own opinion here, but I suspect an evolutionary reasoning for the multitasking/singletasking divide. Men were hunters. Attempting to hunt a wild animal while also reading the paper=dead. Remaining focused on the task without distraction=alive. Fribbler (talk) 13:48, 27 September 2008 (UTC)[reply]

Thank you for your opinions, but can anyone point me to any research on the subjects under discussion? BrainyBabe (talk) 13:53, 27 September 2008 (UTC)[reply]

Hmm, this ones a bit old (2001) and relates to multitasking, but not the internet etc. From the American Psychological Association: [15]. And heres another paper, this time on gender differences and multitasking: [16] Fribbler (talk) 13:58, 27 September 2008 (UTC)[reply]
I'm still not convinced that Media multitasking is always the same as Human multitasking. Your question assumes so, right? For example, I'm currently doing several things that put me into an "atmosphere of information bombardment" but at this moment, I'm fully focused on typing this post. This is not the same as driving while talking on a phone. Zain Ebrahim (talk) 14:28, 27 September 2008 (UTC)[reply]
I've read the research articles (thanks Fribbler) and they seem to find no significant gender difference. I take the point about different sorts of multitasking; looking after children, in particular, has throughout history been combined with other activities. And boring or silent tasks may well be enhanced by another "information stream" -- like oganised reading aloud to monks and nuns at refectory, or those early communists in quiet factories. But these modern shifts in patterns of attention -- the inability to settle down with a book, for example -- and at another level the very existence of the vulgar dismissive phrase TLDR -- these are subjects under discussion, anecdotally, as I said. I suppose it will all come out in the wash (as long as the Blackbery didn't get inadvetently left in the pocket). BrainyBabe (talk) 16:42, 28 September 2008 (UTC)[reply]

an idle speculation about songbirds

"My mother thinks I'm pretty"

Thinking about the raucous squawks of rosellas and peacocks, I suddenly wondered if there's a general rule there: Do beautiful birds have ugly calls and vice versa?

Adambrowne666 (talk) 12:52, 27 September 2008 (UTC)[reply]

You're probably right in general. If the girl is attracted by one thing why invest much trouble in anything else? That's all evolution will do? Dmcq (talk) 13:05, 27 September 2008 (UTC)[reply]
Presumably all birds find some members of their own species attractive. As for what humans find beautiful, most people don't find crows particularly handsome (although I do), and most people would not describe the crow's call as beautiful. I don't know what sound a Marabou stork makes, but I find it hard to imagine it twittering mellifluously. --Sean 15:26, 27 September 2008 (UTC)[reply]
As a general rule (AFAIK) - small birds sing, larger birds squawk. --Kurt Shaped Box (talk) 16:44, 27 September 2008 (UTC)[reply]
Small birds tend to have to hide. Big birds can strut around and be butch, they don't have to sing to get a mate. As to the maribou stork mom thinking its chick is pretty, whilst we're being anthropomorphic perhaps it thinks that's my boy, no nancies in this nest, you'll beat up the neighours in any fight. Dmcq (talk) 23:31, 27 September 2008 (UTC)[reply]
Different birds cue into different things in song depending on the species. Lady prions are able to assess the quality of a male through the call. In smaller songbirds, which have more complex syrnixes, it is song complexity a dn melody that females like, which is why we like them too, but in other species, like bitterns or Kakapos, it is volume or depth. The srceaches of parrots are not mating calls but contact calls to keep in touch with others of the species by the way, and the contact calls (as opposed to mating calls) of songbirds are simpler and less beautiful even in great singers. Sabine's Sunbird talk 22:17, 28 September 2008 (UTC)[reply]
Fair point, SS - but even the mating calls of parrots are (in every species I've ever heard - aside from the Budgerigar, which is melodic in a twittery way) still loud, harsh and screechy/squawky, albeit less so than the ear-splitting contact calls. --Kurt Shaped Box (talk) 22:37, 28 September 2008 (UTC)[reply]

Uranium nitrate in medicine?

My father went to a homeopath and he prescribed my father a liquid medicine(I presume it is a solution) which had the label "uranium nitrate" on it. Could it be this? Is this compound used in medicine? How risky could it be?Leif edling (talk) 15:19, 27 September 2008 (UTC)[reply]

From homeopathy:
Common homeopathic preparations are diluted beyond the point where there is any likelihood that molecules from the original solution are present in the final product; the claim that these treatments still have any pharmacological effect is thus scientifically implausible
The stuff is probably just water, but on the other hand, why drink water from a quack with a uranium supply? --Sean 15:29, 27 September 2008 (UTC)[reply]

You might not have realized this but homeopathy is a popular method of treatment here in India. So is homeopathy totally pseudoscience?Leif edling (talk) 17:21, 27 September 2008 (UTC)[reply]

Yes. The proposed mechanisms are pure bunk. Homeopathy may have some successes via the placebo effect, or simply by providing generally useful suggestions (go for a walk, drink more fluids, prefer natural to processed foods...), but drinking expensive labeled water with no substance in it is no different from drinking plain water with no substance in it. --Stephan Schulz (talk) 17:38, 27 September 2008 (UTC)[reply]
Additionally, I doubt they would be able to treat your father with uranyl nitrate, it's highly toxic, is a hidious onslaught to your kidneys and is likely to be highly regulated, despite the "dilution" that the above quote states. Stick to real, evidence-based medicine, in my opinion. —Cyclonenim (talk · contribs · email) 17:40, 27 September 2008 (UTC)[reply]
If it's homeopathetic then it is statistically highly unlikely to be even one molecule of the "active" ingredient in the treatment...so it's not going to harm you - it's just not going to do you any good either. That's why they can sell the stuff without going through FDA trials, proving efficacy, etc. What you have there is a $100 bottle of water. SteveBaker (talk) 22:03, 27 September 2008 (UTC)[reply]

Perhaps we can put it this way. If it is not diluted to the point of being just water, it is potentially toxic. If it is diluted to the point of being just water, it's just water. --98.217.8.46 (talk) 18:21, 27 September 2008 (UTC)[reply]

I think the point is though that it seems unlikely that any old quack could get ahold of uranyl nitrate. Given this, it seems likely that the quack is a worse quack then your average quack because the quack didn't even follow the rules of quacks Nil Einne (talk) 19:00, 27 September 2008 (UTC)[reply]
The doc test: If it looks like a quack, swims like a quack and ducks like a quack, then it probably isn´t a doc. --Cookatoo.ergo.ZooM (talk) 19:47, 27 September 2008 (UTC)[reply]

Some homeopathic remedies are not completely diluted. But if any of those are actually helpful, it's by pure freakish chance, and not because the homeopaths have any idea what they're doing. Someguy1221 (talk) 22:47, 27 September 2008 (UTC)[reply]

I agree with the general condemnation of homeopathy, but the logic that all poisons are bad is guilty of the same non-scientific over-simplification of which you accuse homeopaths. My mom started her first round of chemotherapy on Friday. Plasticup T/C 05:12, 28 September 2008 (UTC)[reply]
Well, I'm sorry to hear about your mother. I hope she does well. But I don't see where anyone here was arguing at all that all poisons lack medical usage. There is a big difference between saying that homeopathic dilutions are worthless and saying that all "poisons" are medically useless. --98.217.8.46 (talk) 05:27, 28 September 2008 (UTC)[reply]
While you and your mum have my sympathies and I hope it goes well for her, I would say chemotherapy does actually exemplify the point. There's no doubt all chemotherapies are bad for the human body. The only reason we use them is because cancer is often worse and if used effectively, chemotherapy can help to kill the cancerous cells without killing the person who has cancer. In other words, you need a pretty extreme case where administring such a highly toxic compound as uraynl nitrate is worth it unless the uranyl is so dilute to be mostly worthless. Nil Einne (talk) 17:23, 28 September 2008 (UTC)[reply]

hydrogen storage

im trying to find out if i can store hydrogen with low psi for intake injection on my scooter? because i can produce it in house easier. the model h2 toy rocket kit holds at 3 psi before ingnition. would a small compress still heat it to much? could a modifacation be made to the toy rocket generator? —Preceding unsigned comment added by Fearlss69 (talkcontribs) 18:03, 27 September 2008 (UTC)[reply]

If you have a gasoline-powered scooter, it is not advised that you use other fuels, such as hydrogen gas, to operate it. In the first case, the engine will not run very efficiently, if at all. Secondly, gases, like hydrogen, make poor fuels for internal combustion engines. Hydrogen power is FAR more effective and useful in chemical/electrical engines powered by fuel cells. Hydrogen gas works fine for a single ignition rocket engine, which is basically a single controlled burn. For a gasoline-powered internal combustion engine, which depends on carefully timed repeated explosions, changing to a different fuel will mess with that timing, again, if it works at all. If you are asking if it would be a good idea to strap a hydrogen-powered rocket to your scooter, well, via con dios and have a friend with a cell phone who can dial emergency services when you need it. --Jayron32.talk.contribs 20:22, 27 September 2008 (UTC)[reply]
Also, you might just fry your engine due to hydrogen embrittlement. --Stephan Schulz (talk) 20:32, 27 September 2008 (UTC)[reply]
The problem is that if you store raw hydrogen gas at low pressure, you won't have enough of the stuff to go very far. You need to compress the heck out of it to get enough into your tank to be worthwhile. The answer for storing hydrogen is to store it in the form of metal hydrides and react it in a fuel cell to make electricity. Internal combustion simply isn't a smart way to deal with the difficulties of hydrogen. There are other gasses (propane is certainly one) that internal combustion engines handle rather well - but you still need a high pressure feed. SteveBaker (talk) 21:57, 27 September 2008 (UTC)[reply]
The problem with propane is that our atmosphere doesn't seem to like it. Plasticup T/C 05:14, 28 September 2008 (UTC)[reply]
Yep - indeed. But the amount of atmosphere you can make happy by converting an already rather efficient mode of transport to run on hydrogen is pretty limited...and the effort involved with production of said hydrogen in the world as it is today is pretty much guaranteed to cause greater amounts of atmospheric unhappiness than leaving well alone! SteveBaker (talk) 02:42, 29 September 2008 (UTC)[reply]

Recipe for Life

Hydrogen + Ammonia + Methane + Water + Source of Energy + 4 Billion Years = Me!

Is this more or less correct? Sappysap (talk) 18:35, 27 September 2008 (UTC)[reply]

Well you left out a few molecules. See Abiogenesis#Current_models. --98.217.8.46 (talk) 19:53, 27 September 2008 (UTC)[reply]
You should probably include free oxygen - and stipulate 'liquid' water. There are also a bunch of things you should probably specifically exclude (high acidity environment), etc. SteveBaker (talk) 21:52, 27 September 2008 (UTC)[reply]
Electronic Arts + Spore (game) + a few hours = Me! Plasticup T/C 05:16, 28 September 2008 (UTC)[reply]
I don't think this is very correct. Me is more than just base chemicals - you need billions of years of selectional pressures and environmental factors to get humans. Otherwise, well, you might as well say neutrons + protons + electrons + billions of years = Me.--Fangz (talk) 12:51, 28 September 2008 (UTC)[reply]
23/3 (I should probably offer a prize to the first person to correctly explain this answer) SteveBaker (talk) 02:39, 29 September 2008 (UTC)[reply]
You owe me a beer. DMacks (talk) 04:54, 29 September 2008 (UTC)[reply]

Plant species

Could someone help me identify this small plant species? Grows on sandy dunes by the sea. Picture was taken in the summer, in southern Portugal. Very common there. Thanks. Húsönd 21:01, 27 September 2008 (UTC)[reply]

Well, it looks like a succulent, so I will guess it belongs to the Crassulaceae. Although without flowers, I can't really tell for sure what family it belongs to. It grows more like a Sedum than anything else, so it might be Sedum sediforme [17].--Eriastrum (talk) 17:25, 28 September 2008 (UTC)[reply]
Hmm, nope, definitely not the species on the flickr photo. The plants growing on the background of the picture I provided are the same as the one being closed up. It is clear that they're not the same kind as the one appearing on flickr. Yet, they do appear to be Sedum. Húsönd 18:41, 28 September 2008 (UTC)[reply]

September 28

slow step of a reaction

why do we use the slow step of a reaction to determine the rate law of it? —Preceding unsigned comment added by 88.242.107.205 (talk) 00:06, 28 September 2008 (UTC)[reply]

Because it's the bottleneck. See Rate-determining step, specifically the funnel analogy. --Bennybp (talk) 00:34, 28 September 2008 (UTC)[reply]
Try this analogy on for size: Lets say you went to a party with your friends across town. Your buddy lives like 4 houses down from you, and you drive over to his house, leave your car at his house, and drive like 5 miles across town to the party. Now, he ditches you, leaving you to walk home. So, you have to be at work the next day, and need to know when to leave the party to get home, shower, and get to work on time. You have 2 steps to consider: The time to walk 5 miles home, and the time to drive your car the 200 yards or so back to your house. Do you even consider how long the 200 yard drive is going to take? No, because its miniscule compared to the time it will take to walk your half-drunk ass the five miles back to your neighborhood (not to mention the time you will be kicking your friends ass the next time you see him). The point is, in a multi-step process, where each step occurs at a vastly different rate, the only thing you really need to care about is the slowest step. All other steps happen so fast that they don't add appreciably to the overall time for the process. In chemical kinetics, the rate determining step is often multiple orders of magnitude slower (in other words, it can take 1000 times longer or more than any other step), so we only care about the slowest step. The other steps will not effect the overall time of the entire process to within the limits of our measuring devices. So we can safely ignore them. --Jayron32.talk.contribs 01:39, 28 September 2008 (UTC)[reply]

Faster than Light Travel

I was recently in an argument with a friend of mine. I remembered reading somewhere (perhaps in my physics textbook) that if you travel at the speed of light, you stop aging. Travel faster than light and you get younger! Is this information correct? And if so what is the scientific explanation for this? —Preceding unsigned comment added by 203.81.220.111 (talk) 06:28, 28 September 2008 (UTC)[reply]

You should read "A brief history of time" by Stephen Hawking. An oldie but a goodie that addresses a lot of stuff like this. Very enjoyable read. NByz (talk) 08:22, 28 September 2008 (UTC)[reply]


SpacetimeNByz (talk) 08:24, 28 September 2008 (UTC)[reply]
Well first, you can't actually reach the speed of light. See time dilation and the twin paradox for the article and the famous example of phenomenon, respectively. It's not as nifty as it may seem; the person who is aging slower is almost frozen in time as far as the person aging faster can see. So unless you just reeaaallly wanna see the year 3000 AD, it's not terribly useful. Maybe it would work as a very inefficient way of keeping your canned peas good for millenia instead of just years, or a deluxe form of cryopreservation without the cold part... Someguy1221 (talk) 09:23, 28 September 2008 (UTC)[reply]
No, it's not correct. There's a certain restricted sense in which "as you approach the speed of light time goes slower." In the limit of v = c "time stops" (though you can't reach that limit). Some people imagine that when v > c time would go backwards, but that's wrong: the formula for the amount of the slowdown is , which doesn't go negative for v > c. Another thing people sometimes fail to appreciate is that the slow time applies to everything; you age "slower" but also think "slower," so you experience a normal lifespan, not a lengthened one. I put "slower" in quotes because, given that everything slows down equally, it's hard to say what it's slower than. -- BenRG (talk) 10:40, 28 September 2008 (UTC)[reply]
Though if you do have something that is v > c, I seem to recall its worldline does go backwards from certain reference frames, or something like that (like a tachyon). (But I agree this isn't the same thing as saying something is really traveling back in time at all, much less "getting younger".) --98.217.8.46 (talk) 15:11, 28 September 2008 (UTC)[reply]
"Tachyon" is merely a name for some hypothetical thing - they don't exist outside of Star Trek and the math says they can't exist - so let's stop worrying about them. The equation produces the square root of a negative number if v>c -- there are no calculations in the "real world" where the result is a complex number. Even if there were - it's not at all clear what multiplying your age by a complex number would actually mean - but "getting younger" is certainly not it. Essentially, the math fell apart - and that only happens when something impossible is going on. It's a pointless speculation - you can't go as fast as the speed of light if you have any rest mass...so no...just no. SteveBaker (talk) 02:32, 29 September 2008 (UTC)[reply]
Another very dumbed-down way I've heard this explained to non-science guys (so, you science guys can skip this so you don't get worked up by it)... By speed, in this case, you are referring to a ratio between your energy and your mass. To speed up, you can add energy, remove mass, or convert mass to energy. The only way to reach the speed of light is to be 100% energy. Since adding energy will not get rid the mass, the only way to do this is to either remove all mass, leaving only the energy, or to convert all the mass to energy. We do both very well. When heat is emitted by your body, it basically separates itself from your mass and becomes 100% energy. You (the mass) don't get any special benefit from it. Also, if you stand in the middle of a large thermonuclear explosion, your mass will quickly and efficiently be converted to energy. You'll suddenly be travelling at the speed of light. However, you won't be you anymore since there is no means of collecting that energy and putting it back together again to recreate you. So, again, you don't benefit much from the experience. -- kainaw 02:41, 29 September 2008 (UTC)[reply]

Protons and Electrons

Are there equal numbers of protons and electrons in the universe? --220.237.34.141 (talk) 07:32, 28 September 2008 (UTC)[reply]

No. An electron and anti-electron can combine to produce a photon so that's one less of them for a moment for instance. There is charge conservation though. Dmcq (talk) 10:37, 28 September 2008 (UTC)[reply]
Why would the reaction of an electron and an anti-electron be faster (or occur more) than proton + anti-proton?--220.237.34.141 (talk) 11:30, 28 September 2008 (UTC)[reply]
Protons and electrons are really very different things. There's little reason to think that many things will happen the same for both. I suspect there's more electrons than protons, actually - electrons are produced naturally through, for example, pair production. Protons are generally not produced in the same way, because protons are much more massive than electrons and hence require a lot more energy to produce. And lets not get started on virtual particles.--Fangz (talk) 12:47, 28 September 2008 (UTC)[reply]
I wasn't saying such a reaction occurred more often. Simply that if the numbers were equal and this reaction happened then afterwards they would be unequal. Another reaction might make them equal afterwards but overall in the universe even if the numbets are roughly equal then the odd they are actually exactly equal for any length of time is very very very small indeed. Or perhaps you were thinking almost equal rather than equal? You'd then have to specify 'almost' but I suspect the answer even then as Fangz says is no. Dmcq (talk) 13:16, 28 September 2008 (UTC)[reply]
I strongly disagree. If the numbers of protons and electrons were significantly unequal (suppose there were more electrons) - then wouldn't any significant imbalance result in there being a huge negative charge on every macro-scale object? This charge would cause objects to repel each other...and because electromagnetic forces are VASTLY stronger than gravity - wouldn't this cause the immediate spontaneous disintegration of any large objects? The only way I could imagine this NOT happening would be if all of space were uniformly packed with electrons so that all of the repulsions would balance out. But that clearly isn't the case - so why isn't the sun repelling the earth with an ungodly force? No - I think the numbers of protons (or other positively charged particles) must almost exactly equal the number of electrons (or other negatively charged particles)...I don't see how the universe could function otherwise. SteveBaker (talk) 02:24, 29 September 2008 (UTC)[reply]

Einsteins starting point

Where did Einstein start when he formulated his special theory of relativity? For instance, e=mc^2, where did he derive this equation from? Related question, what evidence did he have, if any, that his formulae were correct? How did he know that the direction he was going in was the right one? Cheers 59.100.200.162 (talk) 14:09, 28 September 2008 (UTC)[reply]

This is a big question. You are probably better off getting an actual book about this than asking us to muddle you through the history of it, which is detailed and complicated. We have a longer article on this (e.g. History of special relativity) but in glancing over it right now, it seems fairly unreadable unless you already know all of the physics involved (and is in my opinion historically dubious—it is history the way a scientist tells it, not how a historian tells it, and makes it look like the history is just a bunch of equations being thrown around in a vacuum).
Conceptually the starting point is just the question of how you can reconcile two things Einstein and his contemporaries knew to be true: the speed of light is independent of the speed of its emitter (it is constant in a vacuum), and that Galilean relativity should hold in all physical frame of reference (Galilean relativity is just why you don't feel like you are moving when you are sealed inside the belly of an aircraft, for example, and why you don't feel like you are moving when you are standing on a rapidly rotating planet—in a wholly-enclosed constant frame of reference you cannot know your own absolute speed by simply experiential means). Both of these facts were things that his contemporaries would have found fairly obvious, but wouldn't have necessarily put them together at the same time.
It is in the reconciling of those two facts that most of SR comes out of—if the speed of light is really independent of the speed of its emitter, despite the fact that physical frames of reference will be relative, then two observers traveling at different speeds will measure the speed of light to be the same no matter what their different relative speeds are, and if that is the case, then all sorts of interesting implications come up about time, space, and etc. I won't go through all of those implications here but that's really where it becomes clear that space and time are deeply, deeply linked.
This is a very poor gloss on what Einstein was really doing and really thinking, though. Again, a good historical book (Walter Isaacson's recent biography is fairly good on this point) will illuminate much more clearly this sort of thing. There were a wide variety of factors relating to Einstein's previous education, his work at the patent office in Bern, his meetings with creative friends, the books he read, and so forth.
As for evidence—for much of Einstein's work, there was not any immediate evidence other than the fact that the work was actually conceptually quite simple and the reasoning seemed to work out. But by itself nobody was all that convinced that it was too important or anything other than playing with math. Most of SR was not proven by experimental means until very late in the 20th century, where observations of things like muons made it clear that indeed going very fast can affect size and time. Einstein did not think all of this work was revolutionary—it just made sense. His work on the quanta was much more revolutionary, and garnered much more attention from other scientists. Only after creating General Relativity, and having a few surprising experimental confirmations, did Einstein become known as a very important figure in physics. --98.217.8.46 (talk) 15:06, 28 September 2008 (UTC)[reply]


E=mc² didn't appear in the original paper on relativity, it's from a later paper (though published the same year). There was ample experimental evidence for Einstein's ideas at the time of publication, and in fact all of the math of the theory had already been worked out by Lorentz and Poincaré and others. Einstein's contribution wasn't the math, it was a new philosophical framework for measurement—all the stuff about clocks and metersticks. Prior to 1905 everyone had assumed (without really thinking about it) that there was nothing problematic in principle about synchronizing clocks, though it might be tricky with real, imprecise instruments. Einstein was the first to realize that (a) this was not a philosophically necessary position and (b) it wasn't actually true. I think this was obvious to most physicists once it was pointed out, and the typical reaction to the paper was "of course, how could we be so stupid" and not "that's a strange idea, I wonder if it's true." -- BenRG (talk) 15:12, 28 September 2008 (UTC)[reply]
Well and most of the physicists at the time thought Einstein was just being clever with his reasoning—not that it was really very fundamental work. The recognition that the 1905 papers were something amazing didn't come until later. (Quanta notwithstanding—that was taken up by Planck and others.) --98.217.8.46 (talk) 15:15, 28 September 2008 (UTC)[reply]
Gary Larson suggested that it was actually a custodian who was key to getting the equation we now know. Can't find a copy of the diagram explaining it right now... DMacks (talk) 20:28, 28 September 2008 (UTC)[reply]
I found [18]. PrimeHunter (talk) 02:40, 29 September 2008 (UTC)[reply]

Nightmares

I heard once that being overheated can contribute to the likelihood of nightmares, and it seems to be true for me. Does anyone have information on that? --Masamage 15:31, 28 September 2008 (UTC)[reply]

I've no idea, but I do want to point out that for me, anecdotally, I've noticed that placebo effect on nightmares seems rather significant. I had the idea when I was a kid that if I crossed my arms like a corpse I would have nightmares (hey, I was very young!), and sure enough, as long as I believed it made sense, it worked. When I got old enough to realize how daffy that was, it stopped working. --98.217.8.46 (talk) 15:39, 28 September 2008 (UTC)[reply]
More anecdotes, but overheating -> nightmares does fit a pattern I've noticed. It seems to me that my most vivid dreams come when I'm not deeply asleep; that is, when I'm drifting off or just waking up. Perhaps hypnagogic dreams are an influence there. A more widespread notion is that eating certain foods (like cheese or spicy stuff) can increase bad dreams and that could also contribute to broken sleep. Matt Deres (talk) 16:24, 28 September 2008 (UTC)[reply]
I would say this is one area where it's rather hard to rule out other factors, like the fact that you are 'overheated' means you're probably not going to sleep comfortably, which probably means you're going more likely to remember any dreams, nightmare or otherwise Nil Einne (talk) 17:15, 28 September 2008 (UTC)[reply]
Very true. What may be causing any "overheating" may also be causing nightmares. --Russoc4 (talk) 18:07, 28 September 2008 (UTC)[reply]

Interesting. So, can I take this to mean that physical discomfort in general can increase the chance of having dreams with unpleasant elements? (In the case that's making me ask about it, I think I would have remembered it regardless, because the bad thing in the dream caused me to physically struggle and that's why I woke up.) --Masamage 01:40, 29 September 2008 (UTC)[reply]

Tropical Cyclones sorces

What are other good sources for hurricanes besides the National Hurricane Center's Tropical Cyclone Report.--Leave Message orYellow Evan home 16:11, 28 September 2008 (UTC)[reply]

For Atlantic and East Pacific storms, the TCR is the cat's pajamas. But what sort of information are you looking for? Meteorological data? Damage assessments? Predictions? Social attitudes? Government responses? International aid? Demographic data? Seasonal summaries? Historical trends? Economic impact? Cultural impacts? Impact on immigration? You'll need different sources for all of these things. Plasticup T/C 18:01, 28 September 2008 (UTC)[reply]
A separate but related question: "Cat's pajamas"? That's a new one to me. But I kinda like it.-RunningOnBrains 23:33, 28 September 2008 (UTC)[reply]
The question is whether the cat's pajamas is better than the bee's knees... --Jayron32.talk.contribs 02:25, 29 September 2008 (UTC)[reply]

Let's suppose I have a pencil (or a pen, or a wristwatch, or any small object) and I want to completely disintegrate it, destroying every one of its atoms. Can it (theoretically) be done? If so, how? Is it possible to destroy every atom? Thanks in advance. —Preceding unsigned comment added by XxCutexXxGirlxX (talkcontribs) 17:01, 28 September 2008 (UTC)[reply]

You could get an antimatter pen Nil Einne (talk) 17:14, 28 September 2008 (UTC)[reply]
You could throw it into the sun and let nuclear fusion take its course.
Or you could throw it into a blackhole and allow it to be crushed into a singularity and then evaporated as Hawking radiation. The mass would still exist, but the original atoms would be destroyed forever. APL (talk) 18:55, 28 September 2008 (UTC)[reply]
you could just sit back and wait a few months for the black hole at the LHC to suck it in. 96.231.83.176 (talk) 20:02, 28 September 2008 (UTC)[reply]
  • Throwing it into the Sun won't work. The Sun is only hot enough to fuse hydrogen into helium, not hot enough to destroy all sorts of atoms. (Well, their electrons would be stripped off, but I assume you want the nuclei destroyed.) You need the sort of temperatures found only in a supernova. --Anonymous, 01:35 UTC, September 29, 2008.
A neutron star would be just the thing. None of that hassle with Hawking radiation shooting back the energy of your ex-girlfriend's photo right back at you...forever reminding you...taunting, taunting... SteveBaker (talk) 01:42, 29 September 2008 (UTC)[reply]
You could wait. In a mere 1040 years, all the protons in the pen should have decayed into mostly neutral pions and positrons. Of course, proton decay is only theoretical. If it doesn't exist, you'd have to wait a bit longer (still only 101500 years), at which point all the atoms in the pen should have become iron-56.-RunningOnBrains 23:31, 28 September 2008 (UTC)[reply]
In old science fiction films, the spacemen carried disintegrator guns. Perhaps one of these gadgets could do the job. Edison (talk) 00:17, 29 September 2008 (UTC)[reply]
More seriously, it is theoretically possible (an practically it has been done for some types of atoms) to split large atoms (like metals and carbon) into smaller atoms. If you can do it for one atom, you can do it for each and everyone of them. But you would still have some residual mass of matter. If you want to transform the whole thing into energy, and be left with no mass, then your best bet is antimatter. But why would you want to do that It would require a crazy amount of energy, instead just cuting the link between each of these atoms would make your object puff into various gases without the complexity of spliting the atom --Lgriot (talk) 01:27, 29 September 2008 (UTC)[reply]
Most of the above are impractical and could not be done on earth. Your best chance may be to insert into a atomic accelerator synchrotron or Large hadrion collider and zap it with fast moving nuclei, to break up the nuclei. It will be tough to destroy the protons in the hydrogen, so probably immersion in a nuclear reactor could saturate it with neutrons and covert it to deuterium. Graeme Bartlett (talk) 02:33, 29 September 2008 (UTC)[reply]

Uranium in homeopathic medicine

I asked a related question yesterday and now I've found this: [19]. It's weird; what's with the "make uranium at home" and "homeopathic uranium" statements? Is this the limit of how dangerous homeopathy can get? Leif edling (talk) 17:15, 28 September 2008 (UTC)[reply]

Actually, this sounds almost completely harmless. Crushed "red rock" will bleed negligible amounts of uranium (if any) into the water, and will be quite unlikely to be otherwise harmful. It won't do anything positive, either, but the greatest risk is probably hitting yourself on the thumb while crushing the rock. --Stephan Schulz (talk) 18:01, 28 September 2008 (UTC)[reply]
Furthermore, if the homeopathic "remedy" is prepared "properly", the end user is unlikely to be in any danger; thanks to Avogadro's number, all he's getting is water. (Or whatever they're using as the diluent.) -- Captain Disdain (talk) 01:32, 29 September 2008 (UTC)[reply]
It's really impossible to make sensible statements about something as obviously stupid as homeopathy - but proponents never properly answer any of the most obvious concerns. A sure-fire test for a ridiculous theory is that it typically falls apart even worse when you believe what the nut jobsexperts in the field are claiming. So let's do that...
Let's suspend disbelief for a moment and assume that homeopathy really worked. I mean, really: We're told that the uranium (in this case) left an "imprint" in the water that did something theraputic - and that this effect gets stronger the more you dilute it. My first concern is how come the few silicon atoms that washed off the glass you mixed it in didn't do something? What about the two human skin cells and the million or so bacteria and viruses that inevitably got in there during the process? Water is NEVER 100% pure - so isn't it going to be "imprinted" with countless other things that it came into contact with? The purer the water you start with - the stronger the effect of the impurities are claimed to be - so using super-clean distilled water only makes the 'pollution' in it even more powerful.
This stuff (if you carefully examine the bottles on the shelves at WalMart) is claimed to have a shelf life of at least a year or two - so the "imprinting" in the water molecules has to last at least that long. Where do they get their "non-imprinted" water? There are claims for homeopathic "cures" for low blood pressure and quite different cures for high blood pressure...but how do you make sure that the imprinting for the wrong "cure" wasn't in the water to start with? You really have no clue whether your water came into contact with a (whatever cures low blood pressure) molecule a couple of weeks before you decided to make a high blood pressure cure with it.
What's worse, they claim that the more dilute you make the stuff, the stronger it becomes - but the prior imprinting imposed on the water BEFORE you start your titration is guaranteed to be made yet stronger by your titrations. Since they dilute (say) uranium to the point where statistics and Avagadro's number proves that there is none of the original stuff left, absolutely ANYTHING that was in the water at least as long as a year ago will be having a super-powerful medicinal effect - even if the water is utterly, 100% pure! I imagine that they get their original distilled water from normal sources - so a year ago, your water was probably a part of the ocean - or floating around in a cloud someplace.
In this particular case, you're being asked to take some reddish granite and use that as the starting point "because it contains uranium" - but it's "rock" - it contains hundreds of other kinds of atoms and molecules. If homeopathy really works - then ALL of those things are also being used to "imprint" the water with whatever things they do to you. The "researchers" in this field cannot possibly have tested all of those other molecules in proper human trials - how could they possibly know that your local flavor of granite isn't going to imprint something really nasty into the water?
The claim is always that homeopathic medicines are 100% safe (which they certainly are!) because - for some inadequately explained reason - they only have "beneficial" effects. But how does the underlying chemistry, physics and biology "know" what is "good" and what is "bad". To cure you of a bacterial infection - it has to kill bacteria. How do these homeopathic imprintings "know" to cure the human and not to cure the bacterium?
The whole theory falls apart so seriously under the most gentle nudges from a scientific mind that answering questions about homeopathy is truly impossible to do - other than to state the bloody obvious - which is "IT CANNOT POSSIBLY WORK - EXCEPT BY THE PLACEBO EFFECT". If it does work by the placebo mechanism then why bother with all of the titrations and other bullshit. Just start a new "cult" that says that drinking water from a magic spring will cure you of everything...oh - but wait - the religious nut-jobsexperts already came up with that one.
SteveBaker (talk) 02:13, 29 September 2008 (UTC)[reply]

Answering questions about homeopathy is no different than answering questions about what effecy the witchdoctor's dance is going to have. The basic question is so patently rediculous to anyone that even paid attention in high school that when you say "your just drinking water", people say "no, that can't be it!" and try to find more. Let me restate it. You are just drinking water. If you have a genuine medical concern, find someone who has undergone training in the operation of the human body. If that doesn't work, then ooo eee ooo aaa aaa, zing zang walla walla bing bang... --Jayron32.talk.contribs 02:19, 29 September 2008 (UTC)[reply]

Wow! My splitting headache just went away just by reading that! I'm going to cut and paste it and sell it for $100 and there is nothing you can do to stop me because it's under GFDL! :-) SteveBaker (talk) 03:00, 29 September 2008 (UTC) [reply]
There's nothing particularly mysterious about homeopathy, the placebo effect is well-documented. It seems as though when someone you trust carefully investigates your symptoms and says "take this, it will cure you", in (I think) about 30% of cases above the baseline, the symptoms are alleviated to at least some degree. The homeopathic concept itself is full of crap, but the placebo effect is a mystery that we should be investigating further. Much as I detest the junk spouted about succussion, there is something to the concept of having individual attention paid to one's health. That of course is what homeopaths do, they draw up an individual profile - which of course renders meaningless the prospect of controlled random-double-blind studies - but also renders inaccessible further studies of the placebo effect. Franamax (talk) 03:12, 29 September 2008 (UTC)[reply]
While it may be true that the placebo effect has 30% over baseline positive results, (and I am not sure its that high, but I will accept it axiomatically for the sake of the arguement) any intentional treatment is bound to have much better results. This is especially if your measure for success is that "symptoms are alleviated to at least some degree", since any intentional treatment will already have the placebo effect built in (as long as the patient believes in the cure, the placebo effect should be additive to the theraputic effect of any treatment). While the placebo effect is real and dramatic, its theraputic value is negligible since its effect is true for both real and imagined cures, and the real cures have the added benefit of actually having mechanisms shown to affect the complaint in question... --Jayron32.talk.contribs 03:22, 29 September 2008 (UTC)[reply]

Hydrogen Bonding

Which would require less energy, moving two particles of the same charge close together, or moving two particles of opposite charges close together ? Why? —Preceding unsigned comment added by 24.215.48.99 (talk) 20:25, 28 September 2008 (UTC)[reply]

Visit our Coulomb's law page. DMacks (talk) 20:56, 28 September 2008 (UTC)[reply]
Your question is a bit vague. I can interpret it in any of 3 ways:
  1. You have 2 particles of identical charge that are X distance apart, and 2 particles of identically opposite charge that are also X distance apart. You try to move them against the natural attraction or repulsion of their charges. At the instant you try to move them, which requires more energy. Well, in the first timeless instant when you move either, the answer is they require the same energy BUT...
  2. In the same situation, the two oppositely charged particles will require less and less energy to move because the distance between them (and thus their attraction for each other) is decreasing, BUT the two same charged particles are getting closer, and thus get harder to push. Thus, for any meaningful time interval (that is for any delta-t greater than 0) the same charged particles require more energy.
  3. If you are asking what will happen if you push the particles towards each other in both instances, then the answer is that the same charged particles will require more energy, while the opposite charged particles require NO input of energy (they move spontaneously)... In fact they will generate excess energy equal in magnitude to the energy you had to put into the same-charged system to make them move the same amount (assuming of course no friction).
Hope that helps. --Jayron32.talk.contribs 03:15, 29 September 2008 (UTC)[reply]

dynamics

suppose you have 2 forces pulling a box, the box is resting on a floor with friction, if the box is not moving because all forces are at equilibrium, then do we still need to worry about friction and include it in the calculation? —Preceding unsigned comment added by 142.151.132.11 (talk) 20:34, 28 September 2008 (UTC)[reply]

Is friction a force on the box? If so, does it affect how the box moves when it is pulled? If so, then it is a force that affects the box's motion in response to pulling, and therefore must be included in calculations of that motion and the forces involved. DMacks (talk) 20:48, 28 September 2008 (UTC)[reply]

the static friction is on the box, but my problem is, in which puller's direction is the frictional force going towards? —Preceding unsigned comment added by 142.151.132.11 (talk) 21:00, 28 September 2008 (UTC)[reply]

Friction opposes "motion", so I'd think consider it to oppose whatever motion would be happening otherwise (i.e., opposite the net force of the pullers). DMacks (talk) 21:38, 28 September 2008 (UTC)[reply]
I don't think it changes the result of the calculation, but it could be more complicated than that: if the box is under tension, the friction force could be in both directions on different parts of the bottom of the box. Its sum will be the same as the single force you can calculate instead. --Tardis (talk) 23:09, 28 September 2008 (UTC)[reply]
Since the OP didn't say otherwise, I think it is probably assumed that the box is rigid and you can ignore things like tension. --Tango (talk) 23:27, 28 September 2008 (UTC)[reply]
I imagined that a more correct answer might be at least interesting (to the OP or someone else) if not necessary. It's not entirely irrelevant: after all, for a stiff box the structural distortions associated with the tension and with the friction are of the same general scale. --Tardis (talk) 23:43, 28 September 2008 (UTC)[reply]
In the real world (as opposed to a physics textbook) it could take a larger force to start an object moving than to keep it moving. Edison (talk) 00:14, 29 September 2008 (UTC)[reply]
What you have to do is to resolve all of the non-frictional forces on the box - you can reduce them to a single 'net' force. Now you have an easy problem - is the net force able to overcome friction? What gets double-ikky is that for most real-world surfaces, the 'static' friction is higher than the 'dynamic' friction - so once you've established that the static friction is overcome, you have to do the math again with the dynamic friction in order to determine the final motion of the box. Yukky! SteveBaker (talk) 01:37, 29 September 2008 (UTC)[reply]

September 29

Oceanic Mammalian Question

I reckon that whales, seals, and dolphins have larger lungs than humans that allow them to dive for such long periods of time. However, they also have larger bodies than humans! Do they also have more efficient lungs which better oxygenate their bloodstream? Is that even a real thing?

Thanks! —Preceding unsigned comment added by 76.24.15.197 (talk) 02:33, 29 September 2008 (UTC)[reply]

Our article on whales simply states: "Whales have a unique respiratory system that lets them stay underwater for long periods of time without taking in oxygen." The dolphin article doesn't provide any more insight. -- kainaw 02:45, 29 September 2008 (UTC)[reply]
Dolphins aren't much bigger than humans - their lungs are probably no bigger. I suspect (without more knowledge) that their metabolism is designed such as to allow the animal to continue to function with much lower blood-oxygen levels than humans. But are dolphins really much better at this than humans? Tom Sietas can hold his breath underwater for over 15 minutes (although he cheats a bit by pre-breathing pure oxygen first - his "fair" record is a little over 10 minutes). One assumes that most humans could come close to that ability if they trained to do it since birth as dolphins are. Bottlenosed dolphins can only hold their breath for 30 minutes. So the disparity isn't as great as you might think. SteveBaker (talk) 02:57, 29 September 2008 (UTC)[reply]


MATLAB question

I want to take an arbitrary length array (say, for example, like [1 2 3 4 5]) and convert that into an anonymous function with a function.

So I have a function called any_polynomial which will take an argument of such an array and return the anonymous function, which can be assigned a function handle. This is for an assignment, so I only really want a hint or two. The assignment tells us to use polyval, which is quite useless for this in my opinion, as then the anonymous function will be something like "@(x) polyval(input_array,x)". Thoughts? --M1ss1ontomars2k4 (talk) 03:56, 29 September 2008 (UTC)[reply]

I'm not sure if this is an appropriate question for the science reference desk (it being about Matlab and being homework), but I'll try to help a bit. First of all, I wonder if you haven't misinterpreted the assignment - the function polyval(p,x) evaluates a polynomial at x that has the values p(n) as its nth coefficient. I'm not sure what you mean when you say you want to "convert an array into an anonymous function". I guess what I'm saying is...the anonymous function you have described certainly does something, but it's not clear to me why you don't like it. Could you be more specific about what you want the anonymous function to do? --Bmk (talk) 04:58, 29 September 2008 (UTC)[reply]